CFA各科试题集及解答--QUIZ 6

合集下载

cfa一级考试模拟试题及答案

cfa一级考试模拟试题及答案

cfa一级考试模拟试题及答案CFA一级考试模拟试题及答案1. 以下哪项不是金融市场的基本功能?A. 资源配置B. 风险管理C. 价格发现D. 娱乐消遣答案:D2. 在CFA考试中,以下哪项不属于投资组合管理的三个主要步骤?A. 资产配置B. 证券选择C. 风险管理D. 市场预测答案:D3. 根据现代投资组合理论,以下哪项是影响投资组合预期收益的唯一因素?A. 无风险利率B. 市场风险溢价C. 投资组合的贝塔系数D. 投资组合的波动性答案:C4. 在评估固定收益证券时,以下哪项不是影响债券价格的主要因素?A. 利率水平B. 信用评级C. 债券的到期时间D. 股票市场的波动性答案:D5. 以下哪项不是有效市场假说(EMH)的三种形式之一?A. 弱式有效市场B. 半强式有效市场C. 强式有效市场D. 超级有效市场答案:D6. 在计算股票的自由现金流至企业(FCFF)时,以下哪项不应从营业利润中扣除?A. 折旧和摊销B. 营运资本的变动C. 资本支出D. 净债务的变动答案:C7. 下列关于财务报表分析的说法中,哪项是不正确的?A. 横向分析可以帮助识别趋势B. 纵向分析可以揭示财务比率的变化C. 比率分析可以用于评估公司的财务状况D. 现金流量表是评估公司盈利质量的唯一工具答案:D8. 在评估一个公司的财务健康状况时,以下哪项财务比率不是流动性比率?A. 流动比率B. 速动比率C. 利息保障倍数D. 现金比率答案:C9. 在国际投资中,以下哪项不是影响汇率变动的主要因素?A. 贸易平衡B. 利率差异C. 政治稳定性D. 黄金价格答案:D10. 在CFA考试中,以下哪项不是宏观经济分析的组成部分?A. 经济增长B. 通货膨胀C. 货币政策D. 公司治理答案:D请注意,以上提供的模拟试题及答案仅供参考,实际CFA一级考试的题目和内容可能会有所不同。

考生应以官方提供的教材和学习资料为准备考试的主要依据。

特许金融分析CFA考试试题

特许金融分析CFA考试试题

特许金融分析CFA考试试题1、单选在以下选项中,分析师可以找到有关对厂商的流动性、资本来源和营业结果产生明显不确定影响的信息的是()A、财务报表脚注B、资产负债表和损益表C、管(江南博哥)理者讨论与分析正确答案:C2、单选如果一名消费者愿意按照30万美元的价格购买一栋房屋,但他只需支付24万美元,则上述6万美元的差额为()A、消费者剩余B、消费者赤字C、生产者剩余正确答案:A3、单选假设某股票的B值为1.4,无风险利率为5%,市场组合回报率为8%,该股票的必要回报率是多少()A、9.2%B、10.3%C、15.2%正确答案:A4、单选使基础货币的增长率与实际GDP的长期增长率及目标通货膨胀率之和保持匹配的策略属于()A、通货膨胀目标法B、麦克勒姆规则C、汇率目标法正确答案:B5、单选某新软件的开发者提前收到软件公司的30000美元的预付款。

在收到上述预付款的同时,该开发者支付了8000美元的收益税。

上述软件在下一年才能完成。

试确定该笔预付款在该年的计税基数()A、0美元B、2000美元C、30000美元正确答案:A6、单选在以下因素中,最不可能使投资水平下降的是哪一项因素()A、储蓄率降低B、联邦预算盈余C、资本收益税率上涨正确答案:B7、单选当汽车的平均价格从25000美元下降至23000美元时,某城市的汽车需求数量从3300辆上升至34000辆,则价格需求弹性等于()A、-1.87B、-0.36C、-0.15正确答案:B8、单选当宏观经济处于充分就业的GDP水平时,货币供给的增长最有可能导致的长期影响是()A、更高的产出B、更高的价格C、更低的失业率水平正确答案:B9、单选金融资本最不可能受到以下哪一项的影响?()A、利率B、消费者的收入C、实物资本的MRP正确答案:C10、单选由64名超市顾客组成的随机样本显示顾客的平均消费金额为43美元。

假设该分布符合正态分布,且总体标准差为15美元,则总体平均值的90%的置信区间最接近于()A、41.085~44.915B、40.218~45.782C、39.916~46.084正确答案:C11、问答题特许金融分析师CFA考试科目有哪些?正确答案:①职业标准和操守;②财务报表分析;③量化分析;④经济学参考解析:试题答案①职业标准和操守;②财务报表分析;③量化分析;④经济学;⑤固定收益投资分析;⑥股权分析;⑦投资组合管理;⑧企业金融;⑨衍生工具;⑩其他投资分析等等。

CFA考试试题汇编(含答案)

CFA考试试题汇编(含答案)

CFA考试试题汇编1、The nominal (quoted) annual interest rate on an automobile loan is 10%. The effective annual rate of the loan is 10.47%. The frequency of compounding periods per year for the loan is closest to:【单选题】A.weekly.B.monthly.C.quarterly.正确答案:B点拨::“The Time Value of Money,” Richard A. DeFusco, CFA, Dennis W. McLeavey, CFA, Jerald E. Pinto, CFA, and David E. Runkle, CFA2013 Modular Level I, Vol. 1, Reading 5, Section 3.3Study Session 2–5–c, dCalculate and interpret the effective annual rate, given the stated annual interest rate and the frequency of compounding. Solve time value of money problems for different frequencies of compounding:B is correct. Use the formula for effective annual rate:Iteratively substitute the possible frequency of compounding until the EAR is 10.47%.Thus, the correct answer is monthly compounding.2、Which of the following is a constraint as defined in the International Financial Reporting Standards (IFRS) Framework for the Preparation and Presentation of Financial Statements?【单选题】A.NeutralityB.TimelinessC.Going concern正确答案:B点拨::“Financial Reporting Standards,” Thomas R. Robinson, CFA, Jan Hendrik van Greuning, CFA, Karen O’Connor Rubsam, CFA, R. Elaine Henry, CFA, and Michael A. Broihahn, CFATimeliness is a constraint in the IFRS Framework. Neutrality is a factor that contributes to reliability and going concern is an assumption of the Framework.3、Which method of calculating the firm’s cost of equity is most likely to incorporate the long-run return relationship between the firm's stock and the market portfolio?【单选题】A.Dividend discount modelB.Capital asset pricing modelC.Bond-yield-plus risk-premium正确答案:B点拨::“Cost of Capital” Yves Courtois, CFA, Gene C. Lai, and Pamela Peterson Drake, CFAThe capital asset pricing model uses the firm’s equity beta, which is computed from a market model regression of the company's stock returns against market returns.4、For a 90-day U.S. Treasury bill selling at a discount, which of the following methods most likely results in the highest yield?【单选题】A.Money market yieldB.Discount-basis yieldC.Bond equivalent yield正确答案:C点拨::“Working Capital Management,” Edgar Norton, Jr., Kenneth L. Parkinson, and Pamela Peterson Drake5、An investor gathers the following data.To estimate the stock's justified forward P/E, the investor prefers to use the compounded annual earnings growth and the average of the payout ratios over the relevant period (i.e., 2008–2011). If the investor uses 11.5% as her required rate of return, the stock's justified forward P/E is closest to:【单选题】A.10.B.12.C.21.正确答案:C点拨::“Equity Valuation: Concepts and Basic Tools,” John J. Nagorniak and Stephen E.Wilcox6、A bond portfolio manager is considering three Bonds – A, B, and C – for his portfolio. Bond A allows the issuer to call the bond before stated maturity, Bond B allows the investor to put the bond back to the issuer before stated maturity, and Bond C contains no embedded options. The bonds are otherwise identical. The manager tells his assistant, “Bond A and Bond B should have larger nominal yield spreads to a U.S. Treasury than Bond C to compensate for their embedded options.”Is the manager most likely correct?【单选题】A.Yes.B.No, Bond A’s nominal yield spread should be less than Bond C’s.C.No, Bond B’s nominal yield spread should be less than Bond C’s.正确答案:C点拨::“Understanding Yield Spreads,” Frank J. Fabozzi, CFAC is correct because Bond B’s embedded put option benefits the investor and the yield spread willtherefore be less than the yield spread of Bond C, which does not contain this benefit.7、Which of the following characteristics is best described that the information in financialstatements can influence user's economic decisions or affect user's evaluationof past events or forecasts of future events in accordance with the IFRSframework's definitions and recognition criteria?【单选题】A.Relevance.parability.C.Faithful representation.正确答案:A点拨::根据IFRS的条款,财务报表的两个基本特性使得这些财务信息有用,这两个特性包括相关性(relevance)和公允陈述(faithful representation)。

cfa一级考试题库答案

cfa一级考试题库答案

cfa一级考试题库答案
1. 以下哪项是CFA一级考试中关于财务报表分析的主要内容?
A. 理解财务报表的结构和内容
B. 识别和分析财务报表中的异常情况
C. 预测未来财务报表的趋势
D. 所有以上选项
正确答案:D. 所有以上选项
2. 在CFA一级考试中,关于投资组合管理的哪项说法是正确的?
A. 投资组合管理只关注资产配置
B. 投资组合管理包括资产配置、选择证券和风险管理
C. 投资组合管理与市场时机无关
D. 投资组合管理只涉及股票投资
正确答案:B. 投资组合管理包括资产配置、选择证券和风险管理
3. 在CFA一级考试中,关于固定收益证券的以下哪项说法是错误的?
A. 固定收益证券的利息支付是固定的
B. 固定收益证券的价格与市场利率呈负相关
C. 固定收益证券的风险高于股票
D. 固定收益证券的到期收益率是固定的
正确答案:C. 固定收益证券的风险高于股票
4. 在CFA一级考试中,关于衍生品的以下哪项说法是正确的?
A. 衍生品的价值完全独立于标的资产
B. 衍生品可以用来对冲风险
C. 衍生品只能用于投机
D. 衍生品不能用于投资组合管理
正确答案:B. 衍生品可以用来对冲风险
5. 在CFA一级考试中,关于经济学的以下哪项说法是错误的?
A. 经济学研究资源的分配和利用
B. 宏观经济学关注整体经济的运行
C. 微观经济学研究个体经济单位的行为
D. 经济学只关注市场均衡状态
正确答案:D. 经济学只关注市场均衡状态
结束语:以上是CFA一级考试题库中的一些典型题目及其答案,希望能够帮助考生更好地准备考试。

2019年CFA一级notes整理数量分析习题

2019年CFA一级notes整理数量分析习题

数量分析习题Los6MODULE QUIZ 6.1To best evaluate your performance,enter your quiz answers online.1.An interest rate is best interpreted as:A.a discount rate or a measure of risk.B.a measure of risk or a required rate of return.C.a required rate of return or the opportunity cost of consumption.2.An interest rate from which the inflation premium has been subtracted is known as:A.a real interest rate.B.a risk-free interest rate.C.a real risk-free interest rate.3.What is the effective annual rate for a credit card that charges18%compounded monthly?A.15.38%.B.18.81%.C.19.56%.4.An investor is looking at a$150,000home.If20%must be put down and the balance is financed at9%over thenext30years,what is the monthly mortgage payment?A.$799.33.B.$895.21.C.$965.55.5.Given daily compounding,the growth of$5,000invested for one year at12%interest will be closest to:A.$5,600.B.$5,628.C.$5,637.MODULE QUIZ 6.2To best evaluate your performance,enter your quiz answers online.1.The amount an investor will have in15years if$1,000 is invested today at an annual interest rate of9%will be closest to:A.$1,350.B.$3,518.C.$3,642.2.How much must be invested today,at8%interest,to accumulate enough to retirea$10,000debt due seven years from today?A.$5,835.B.$6,123.C.$8,794.3.An investor has just won the lottery and will receive $50,000per year at the end of each of the next20years. At a10%interest rate,the present value of thewinnings is closest to:A.$425,678.B.$637,241.C.$2,863,750.4.An investor is to receive a15-year,$8,000annuity, with the first payment to be received today.At an11% discount rate,this annuity’s worth today is closest to:A.$55,855.B.$57,527.C.$63,855.5.If$1,000is invested today and$1,000is invested at the beginning of each of the next three years at12% interest(compounded annually),the amount an investor will have at the end of the fourth year will be closest to:A.$4,779.B.$5,353.C.$6,792.6.Terry Corporation preferred stocks are expected to pay a$9annual dividend forever.If the required rate of return on equivalent investments is11%,a shareof Terry preferred should be worth:A.$81.82.B.$99.00.C.$122.22.MODULE QUIZ 6.3To best evaluate your performance,enter your quiz answers online.1.An analyst estimates that XYZ’s earnings will grow from$3.00a share to$4.50pershare over the next eight years.The rate of growth in XYZ’s earnings is closest to:A. 4.9%.B. 5.2%.C. 6.7%.2.If$5,000is invested in a fund offering a rate of return of12%per year,approximately how many years will it take for the investment to reach$10,000?A.4years.B.5years.C.6years.3.An investment is expected to produce the cash flows of $500,$200,and$800atthe end of the next three years.If the required rate of return is12%,the presentvalue of this investment is closest to:A.$835.B.$1,175.C.$1,235.4.If$10,000is invested today in an account that earns interest at a rate of9.5%,what is the value of the equal withdrawals that can be taken out of the account atthe end of each of the next five years if the investor plans to deplete the accountat the end of the time period?A.$2,453.B.$2,604.C.$2,750.5.Given an11%rate of return,the amount that must be put into an investment account at the end of each of thenext10years in order to accumulate$60,000to pay for a child’s education is closest to:A.$2,500.B.$3,588.C.$4,432.6.An investor will receive an annuity of$4,000a year for10years.The first payment is to be received five years from today.At a9%discount rate,this annuity’s worth today is closest to:A.$16,684.B.$18,186.C.$25,671.MODULE QUIZ7.1To best evaluate your performance,enter your quiz answers online.1.A company is considering entering into a joint venture that will require an investment of$10million.The project is expected to generate cash flows of$4million, $3million,and$4million in each of the next three years, respectively.Assuming a discount rate of10%,what is the project’s NPV?A.–$879,000.B.–$309,000.C.+$243,000.2.Goodeal,Inc.,is considering the purchase of a new material handling system for a cost of$15million.This system is expected to generate a positive cash flow of $1.8million per year in perpetuity.What is the NPV of the proposed investment if the appropriate discount rate is10.5%?A.$2,142,857.B.$13,200,000.C.$17,142,857.3.Goodeal,Inc.,is considering the purchase of a new material handling system for a cost of$15million.This system is expected to generate a positive cash flow of $1.8million per year in perpetuity.What is the IRR of the proposed investment if the appropriate hurdle rate is10.5%?A.8.3%.B.10.5%.Video coveringthis content isavailable online.C.12.0%.4.Which of the following statements least accurately describes the IRR and NPV methods?A.The discount rate that gives an investment an NPV of zero is theinvestment’s IRR.B.If the NPV and IRR methods give conflicting decisions for mutually exclusive projects,the IRR decision should be used to select the project.C.The NPV method assumes that a project’s cash flows will be reinvested atthe cost of capital,while the IRR method assumes they will be reinvested atthe IRR.5.Which of the following statements least accurately describes the IRR and NPV methods?A.A project’s IRR can be positive even if the NPV is negative.B.A project with an IRR equal to the cost of capital will have an NPV of zero.C.A project’s NPV may be positive even if the IRR is less than the cost of capital.6.What should an analyst recommend based on the following information for two independent projects?A.Accept A and rejectB.B.Reject A and accept B.C.Accept A and accept B.7.An investor buys a stock for$40.The stock pays a dividend of$0.75in Year1and$1.25in Year 2.At the end of Year2the investor sells the stock for$44.What is the investor’s holding period return?A.7.5%.B.10.0%.C.15.0%.MODULE QUIZ7.2To best evaluate your performance,enter your quiz answers online.1.Which of the following statements least likely represents a characteristic of the time-weighted rate of return?It is:A.not affected by the timing of cash flows.ed to measure the compound rate of growth of$1over a stated measurement period.C.defined as the internal rate of return on an investment portfolio,taking into account all inflows and outflows. Use the following data to answer Questions2and 3.Assume an investor purchases a share of stock for$50at time t=0,and another share at$65at time t=1,and at the end of Year1and Year2,the stock paid a$2dividend. Also,at the end of Year2,the investor sold both shares for$70each.2.The money-weighted rate of return on the investment is:A.15.45%.B.16.73%.C.18.02%.3.The time-weighted rate of return on the investment is:A.18.27%.B.20.13%.C.21.83%.4.What is the bank discount yield for a T-bill that is selling for$99,000,with a face value of$100,000and95 days remaining until maturity?A. 1.51%.B. 3.79%.C. 6.00%.5.What is the holding period yield for a T-bill that is selling for$99,000if it has a face value of$100,000and 95days remaining until maturity?A. 1.01%.B. 2.03%.C. 3.79%.6.What is the effective annual yield for a T-bill that is selling for$99,000if it has a face value of$100,000and 95days remaining until maturity?A. 3.79%.B. 3.94%.C. 4.50%.7.What is the money market yield for a T-bill that is selling for$99,000if it has a face value of$100,000and 95days remaining until maturity?A. 3.79%.B. 3.83%.C. 3.90%.8.Which of the following is least accurate regarding a bank discount yield?A.It ignores the opportunity to earn compound interest.B.It is based on the face value of the bond,not itspurchase price.C.It reflects the nonannualized return an investor will earn over a security’s life.9.A175-day T-bill has an effective annual yield of 3.80%. Its money market yield is closest to:A. 1.80%.B. 3.65%.C. 3.71%.MODULE QUIZ8.1To best evaluate your performance,enter your quiz answers online.1.In the card game of poker,winning hands are determined by rank(high card,pair,two pair,etc.).The scale of poker hand rankings is:A.a nominal scale.B.an ordinal scale.C.an interval scale.2.The intervals in a frequency distribution should always have which of the following characteristics?The intervals should always:A.be truncated.B.be open ended.C.be nonoverlapping.Use the following frequency distribution for Questions3 through5.3.The number of intervals in this frequency table is:A. 1.B. 5.C.16.4.The sample size is:A. 1.B. 5.C.16.5.The relative frequency of the second interval is:A.10.0%.B.16.0%.C.43.8%.6.The vertical axis of a histogram shows:A.the frequency with which observations occur.B.the range of observations within each interval.C.the intervals into which the observations are arranged. MODULE QUIZ8.2To best evaluate your performance,enter your quiz answers e the following data to answer Questions1through9.XYZ Corp.Annual Stock Returns1.What is the arithmetic mean return for XYZ stock?A.7.3%.B.8.0%.C.11.0%.2.What is the median return for XYZ stock?A.7.3%.B.8.0%.C.11.0%.3.What is the mode of the returns for XYZ stock?B.8.0%.C.11.0%.4.What is the range for XYZ stock returns?A.11.0%.B.22.0%.C.29.0%.5.What is the mean absolute deviation for XYZ stock returns?A. 5.20%.B.7.33%.C.29.0%.6.Assuming that the distribution of XYZ stock returns isa population,what is the population variance?A. 6.8%2.B.7.7%2.C.80.2%2.7.Assuming that the distribution of XYZ stock returns isa population,what is the population standard deviation?A. 5.02%.B.8.96%.C.46.22%.8.Assuming that the distribution of XYZ stock returns isa sample,the sample variance is closest to:A. 5.0%2.B.72.4%2.C.96.3%2.9.Assuming that the distribution of XYZ stock returns isa sample,what is the sample standard deviation?A.9.8%.B.72.4%.C.96.3%.Use the following data to answer Questions10and11.The annual returns for FJW’s common stock over the years2013,2014,2015,and2016were15%,19%,–8%,and14%. 10.What is the arithmetic mean return for FJW’s common stock?A.10.00%.B.14.00%.C.15.25%.11.What is the geometric mean return for FJW’s common stock?A.9.45%.B.14.21%.C.It cannot be determined because one of the returns is negative.12.The harmonic mean of3,4,and5is:A. 3.74.C. 4.12.13.Given the following observations:2,4,5,6,7,9,10,11The65th percentile is closest to:A. 5.85.B. 6.55.C.8.70.MODULE QUIZ8.3To best evaluate your performance,enter your quiz answers online.1.For a skewed distribution,what is the minimum percentage of the observations that will lie between±2.5standard deviations of the mean based on Chebyshev’s Inequality?A.56%.B.75%.C.84%.2.A portfolio has had a mean return of 6.0%with a standard deviation of 2.5%.The risk-free rate is 1.0%and inflation was 1.5%.The portfolio’s Sharpe ratio is closest to:A.0.5.B. 1.0.C. 2.0.3.A distribution of returns that has a greater percentage of small deviations from the mean and a greater percentage of extremely large deviations from the mean compared to a normal distribution:A.is positively skewed.B.has positive excess kurtosis.C.has negative excess kurtosis.4.Which of the following is most accurate regarding a distribution of returns that has a mean greater than its median?A.It is positively skewed.B.It is a symmetric distribution.C.It has positive excess kurtosis.5.A normal distribution has kurtosis of:A.zero.B.one.C.three.6.The most appropriate measure of central tendency for forecasting an investment return in a single future period is:A.the harmonic mean.B.the arithmetic mean.C.the geometric mean.MODULE QUIZ9.1To best evaluate your performance,enter your quiz answers online.1.An event that includes all of the possible outcomes is said to be:A.random.B.exclusive.C.exhaustive.2.Which of the following values cannot be the probability of an event?A.0.00.B. 1.00.C. 1.25.3.The probability that the DJIA will increase tomorrow is 2/3.The probability of an increase in the DJIA stated as odds is:A.two-to-one.B.one-to-three.C.two-to-three.4.The multiplication rule of probability determines the joint probability of two events as the product of:A.two conditional probabilities.B.two unconditional probabilities.C.a conditional probability and an unconditional probability.5.If events A and B are mutually exclusive,then:A.P(A|B)=P(A).B.P(AB)=P(A)×P(B).C.P(A or B)=P(A)+P(B).6.At a charity ball,800names are put into a hat.Four of the names are identical.Ona random draw,what is the probability that one of these four names will be drawn?A.0.004.B.0.005.C.0.010.7.Two mutually exclusive events:A.always occur together.B.cannot occur together.C.can sometimes occur together.MODULE QUIZ9.2To best evaluate your performance,enter your quiz answers online.1.Two events are said to be independent if the occurrence of one event:A.means that the second event cannot occur.B.means that the second event is certain to occur.C.does not affect the probability of the occurrence of the other event.2.An analyst estimates that a share price has an80% probability of increasing if economic growth exceeds3%,a 40%probability of increasing if economic growthis positive but less than3%,and a10%probability of increasing if economic growth is negative.If economic growth has a25%probability of exceeding3%and a25%probability of being negative,what is the probability that the share price increases?B.42.5%.C.62.5%.3.Possible values for the covariance of two random variables are:A.unbounded.B.bounded by0and 1.C.bounded by–1and 1.MODULE QUIZ9.3To best evaluate your performance,enter your quiz answers online.1.Given the conditional probabilities in the table below and the unconditional probabilities P(Y=1)=0.3and P(Y =2)=0.7,what is the expected value of X?A. 5.0.B. 5.3.C. 5.7.2.A discrete uniform distribution(each event has an equal probability of occurrence)has the following possible outcomes for X:[1,2,3,4].The variance of this distribution is closest to:A. 1.00.B. 1.25.C. 2.00.3.The correlation of returns between Stocks A and B is 0.50.The covariance between these two securities is0.0043,and the standard deviation of the return of StockB is26%.The variance of returns for Stock A is:A.0.0011.B.0.0331.C.0.2656.4.An analyst believes Davies Company has a40% probability of earning more than$2per share.She estimates that the probability that Davies Company’s credit rating will be upgraded is70%if its earnings per share are greater than$2and20%if its earnings per share are$2or less.Given the information that DaviesCompany’s credit rating has been upgraded,what is the updated probability that its earnings per share are greater than$2?A.50%.B.60%.C.70%.5.Consider a universe of10bonds from which an investor will ultimately purchase six bonds for his portfolio.If the order in which he buys these bonds is not important, how many potential6-bond combinations are there?A.7.C.5,040.6.There are10sprinters in the finals of a race.How many different ways can the gold,silver,and bronze medals be awarded?A.120.B.720.C.1,440.MODULE QUIZ10.1To best evaluate your performance,enter your quiz answers online.1.Which of the following is least likely an example of adiscrete random variable?A.The number of stocks a person owns.B.The time spent by a portfolio manager with a client.C.The number of days it rains in a month in Iowa City.2.For a continuous random variable X,the probability of any single value of X is:A.one.B.zero.C.determined by the cdf.4.The probability that X=3is:A.0.18.B.0.24.C.0.43.5.The cdf of5,or F(5)is:A.0.17.B.0.71.C.0.88.6.The probability that X is greater than3is:A.0.24.B.0.43.C.0.67.7.What is P(2≤X≤5)?A.0.17.B.0.38.C.0.73.8.The expected value of the random variable X is:A. 3.35.B. 3.70.C. 5.47.9.Which of the following is least likely a condition of a binomial experiment?A.There are only two trials.B.The trials are independent.C.If p is the probability of success,and q is the probability of failure,then p+q= 1.10.Which of the following statements least accurately describes the binomial distribution?A.It is a discrete distribution.B.The probability of an outcome of zero is zero.C.The combination formula is used in computing probabilities.11.A recent study indicated that60%of all businesses have a fax machine.From the binomial probabilitydistribution table,the probability that exactly four businesses will have a fax machine in a random selection of six businesses is:A.0.138.B.0.276.C.0.311.12.Ten percent of all college graduates hired stay with the same company for more than five years.In a random sample of six recently hired college graduates,the probability that exactly two will stay with the same company for more than five years is closest to:A.0.098.B.0.114.C.0.185.13.Assume that40%of candidates who sit for the CFA®examination pass it the first time.Of a random sample of 15candidates who are sitting for the exam for thefirst time,what is the expected number of candidates that will pass?A.0.375.B. 4.000.C. 6.000.14.A continuous uniform distribution has the parameters a =4and b=10.The F(20)is:A.0.25.B.0.50.C. 1.00.MODULE QUIZ10.2To best evaluate your performance,enter your quiz answers online.1.A key property of a normal distribution is that it:A.has zero skewness.B.is asymmetrical.C.has zero kurtosis.2.Which of the following parameters is necessary to describe a multivariate normal distribution?A.Beta.B.Correlation.C.Degrees of freedom.Use the following data to answer Questions3through 5.A study of hedge fund investors found that their annual household incomes are normally distributed with a mean of $175,000and a standard deviation of$25,000.3.The percentage of hedge fund investors that haveincomes less than$100,000is closest to:A.0.05%.B.0.10%.C.0.13%.4.The percentage of hedge fund investors that have incomes greater than$225,000is closest to:A.0.50%.B. 1.10%.C. 2.28%.5.The percentage of hedge fund investors that have incomes greater than$150,000is closest to:A.34.13%.B.68.26%.C.84.13%.6.For the standard normal distribution,the z-value gives the distance between the mean and a point in terms of:A.the variance.B.the standard deviation.C.the center of the curve.7.For a standard normal distribution,F(0)is:A.0.0.B.0.1.C.0.5.MODULE QUIZ10.3To best evaluate your performance,enter your quiz answers online.1.Given a threshold level of return of4%,use Roy’s safety-first criterion to choose the optimal portfolio. Portfolio:A. A.B. B.C. C.2.Given a threshold level of return of0%,use Roy’s safety-first criterion to choose the optimal portfolio. Portfolio:A. A.B. B.C. C.3.Portfolio A has a safety-first ratio of 1.3with a threshold return of2%.What is the shortfall risk for a threshold return of2%?A.9.68%.B.40.30%.C.90.30%.4.For a lognormal distribution:A.the mean equals the median.B.the probability of a negative outcome is zero.C.the probability of a positive outcome is50%.5.If a stock’s initial price is$20and its year-end price is$23,then its continuously compounded annual rate of return is:A.13.64%.B.13.98%.C.15.00%.6.A stock doubled in value last year.Its continuously compounded return over the period was closest to:A.18.2%.B.69.3%.C.100.0%.ing hypothesized parameter values and a random number generator to study the behavior of certain asset returns is part of:A.historical analysis.B.Monte Carlo simulation.C.standardizing a random variable.MODULE QUIZ11.1To best evaluate your performance,enter your quiz answers online.1.A simple random sample is a sample drawn in such a way that each member of the population has:A.some chance of being selected in the sample.B.an equal chance of being included in the sample.C.a1%chance of being included in the sample.2.The sampling distribution of a statistic is the probability distribution made up of all possible:A.observations from the underlying population.B.sample statistics computed from samples of varying sizes drawn from thesame population.C.sample statistics computed from samples of the same size drawn from thesame population.3.Sampling error is defined as:A.an error that occurs when a sample of less than30 elements is drawn.B.an error that occurs during collection,recording,and tabulation of data.C.the difference between the value of a sample statisticand the value of the corresponding population parameter. 4.The mean age of all CFA candidates is28years.The mean age of a random sample of100candidates is found to be26.5years.The difference of 1.5years is called:A.the random error.B.the sampling error.C.the population error.5.The sample of debt/equity ratios of25publicly traded U.S.banks as of fiscal year-end2003is an example of:A.a point estimate.B.cross-sectional data.C.a stratified random sample.6.To apply the central limit theorem to the sampling distribution of the sample mean,the sample is usually considered to be large if n is greater than:A.20.B.25.C.30.7.If n is large and the population standard deviation is unknown,the standard error of the sampling distribution of the sample mean is equal to:A.the sample standard deviation divided by the samplesize.B.the population standard deviation multiplied by the sample size.C.the sample standard deviation divided by the square root of the samplesize.8.The standard error of the sampling distribution of the sample mean for a sample size of n drawn from a population with a mean ofµand a standard deviation ofσis:A.sample standard deviation divided by the sample size.B.sample standard deviation divided by the square root of the sample size.C.population standard deviation divided by the square root of the samplesize.9.Assume that a population has a mean of14with a standard deviation of 2.If a random sample of49 observations is drawn from this population,the standard error of the sample mean is closest to:A.0.04.B.0.29.C. 2.00.10.The population’s mean is30and the mean of a sample of size100is28.5.The variance of the sample is25.The standard error of the sample mean is closest to: A.0.05.C.0.50.11.Which of the following is least likely a desirable property of an estimator?A.Reliability.B.Efficiency.C.Consistency.MODULE QUIZ11.2To best evaluate your performance,enter your quiz answers online.1.Which of the following is least likely a property of Student’s t-distribution?A.As the degrees of freedom get larger,the variance approaches zero.B.It is defined by a single parameter,the degrees of freedom,which is equalto n–1.C.It has more probability in the tails and less at the peak than a standardnormal distribution.2.A random sample of100computer store customers spent an average of$75at the store.Assuming the distribution is normal and the population standard deviation is$20, the95%confidence interval for the population mean is closest to:A.$71.08to$78.92.B.$73.89to$80.11.C.$74.56to$79.44.3.Best Computers,Inc.,sells computers and computer parts by mail.A sample of25recent orders showed the mean time taken to ship out these orders was70hours with a sample standard deviation of14hours.Assuming the population is normally distributed,the99%confidence interval for the population mean is:A.70±2.80hours.B.70±6.98hours.C.70±7.83hours.4.What is the most appropriate test statistic for constructing confidence intervals for the population mean when the population is normally distributed,but the variance is unknown?A.The z-statistic atαwith n degrees of freedom.B.The t-statistic atα/2with n degrees of freedom.C.The t-statistic atα/2with n–1degrees of freedom.5.When constructing a confidence interval for the population mean of a nonnormal distribution when the population variance is unknown and the sample size islarge(n>30),an analyst may acceptably use:A.either a z-statistic or a t-statistic.B.only a z-statistic atαwith n degrees of freedom.C.only a t-statistic atα/2with n degrees of freedom.6.Jenny Fox evaluates managers who have a cross-sectional population standard deviation of returns of8%.If returns are independent across managers,howlarge of a sample does Fox need so the standard error of sample means is1.265%?A.7.C.40.7.Annual returns on small stocks have a population mean of12%and a population standard deviation of20%.If the returns are normally distributed,a90%confidence interval on mean returns over a5-year period is:A. 5.40%to18.60%.B.–2.75%to26.75%.C.–5.52%to29.52%.8.An analyst who uses historical data that was not publicly available at the time period being studied will have a sample with:A.look-ahead bias.B.time-period bias.C.sample selection bias.9.Which of the following is most closely associated with survivorship bias?A.Price-to-book studies.B.Stratified bond sampling studies.C.Mutual fund performance studies.MODULE QUIZ12.1To best evaluate your performance,enter your quiz answers online.1.To test whether the mean of a population is greater than20,the appropriate null hypothesis is that the population mean is:A.less than20.B.greater than20.C.less than or equal to20.2.Which of the following statements about hypothesis testing is most accurate?A.A Type II error is rejecting the null when it is actually true.B.The significance level equals one minus the probability of a Type I error.C.A two-tailed test with a significance level of5%has z-critical values of±1.96.3.For a hypothesis test with a probability of a Type II error of60%and a probability of a Type I error of5%, which of the following statements is most accurate?A.The power of the test is40%,and there is a5% probability that the test statistic will exceed the critical value(s).B.There is a95%probability that the test statistic will be between the critical values if this is a two-tailed test.C.There is a5%probability that the null hypothesis will be rejected when actually true,and the probability of rejecting the null when it is false is40%.4.If the significance level of a test is0.05and the probability of a Type II error is0.15,what is the power of the test?A.0.850.B.0.950.C.0.975.MODULE QUIZ12.2To best evaluate your performance,enter your quiz answers online.ing historical data,a hedge fund manager designs a test of whether abnormalreturns are positive on average.The test results in a p-value of3%.The manager can most appropriately:A.reject the hypothesis that abnormal returns are less than or equal to zero,using a1%significance level.B.reject the hypothesis that abnormal returns are less than or equal to zero,using a5%significance level.C.conclude that the strategy produces positive abnormal returns on average,using a5%significance level.2.An analyst wants to test a hypothesis concerning the population mean of monthly returns for a composite that has existed for24months.The analyst may appropriately。

2023年CFA一级考试真题与解析

2023年CFA一级考试真题与解析

2023年CFA一级考试真题与解析2023年CFA一级考试已经圆满结束,本文将为大家带来这次考试的真题与解析,帮助考生更好地了解考试内容和备考要点。

第一部分:道德与职业准则第一题:伦理与个人行为题目:小林是一位注册投资顾问,他的新客户Mr. Smith希望通过投资来增加自己的财富。

小林了解到Mr. Smith退休计划的绝大部分资金已经投资在一个过度集中的地方,并且其他重要的因素也没有得到考虑。

这种情况下,小林应该如何处理?解析:根据CFA协会的道德与职业准则,投资顾问有责任了解客户的财务情况和投资目标,并提供客观、全面的建议。

在这种情况下,小林应该提醒客户他所面临的风险,并提出分散投资、考虑其他因素的建议,以保护客户的财务利益。

第二题:外部投资者关系题目:公司XYZ近期在一家领先的社交媒体平台上展开一项新产品的在线推广活动。

按照与CFA协会的协议,公司XYZ应如何披露这个推广活动?解析:根据CFA协会的投资者关系准则,公司在展开新产品的推广活动时,需要向投资者提供及时、准确的信息。

公司XYZ应在适当的时候披露这个推广活动,提供项目的重要信息以及潜在的风险和影响,以便投资者做出明智的决策。

第二部分:投资工具第三题:权益估值模型题目:一个分析师使用股利贴现模型(Dividend Discount Model,DDM)来估算一家公司的股票价格。

该公司估计未来一年的股息为2美元,预计增长率为5%,使用12%的折现率。

根据这些数据,该公司的股票价格为多少?解析:根据股利贴现模型,股票价格等于未来股息现值的总和。

在这个例子中,未来一年的股息为2美元,增长率为5%,折现率为12%。

根据计算,未来股息的现值为2/(0.12-0.05)=33.33美元。

因此,该公司的股票价格为33.33美元。

第四题:债券估值题目:一家公司发行了一笔面值为1000美元、票面利率为6%、到期日为5年的债券。

目前市场利率为8%。

根据这些信息,该债券的市场价格是多少?解析:债券的市场价格等于未来现金流的现值之和。

CFA考试试题汇编(含答案)[完整版]

CFA考试试题汇编(含答案)[完整版]

CFA考试试题汇编(含答案)[完整版]1. Which of the following statements about the CFA Institute's Professional Conduct Program (PCP) is least accurate?A. Possible sanctions include condemnation by a member's peers or suspension of a candidate's participation in the CFA Program.B. If the Designated Officer determines that a sanction against a member is warranted, the member must either accept the sanction or lose the right to use the CFA designation.C. Members who cooperate with a PCP inquiry by providing confidential client information to PCP staff are not in violation of Standard III(E) Preservation of Confidentiality.【答案】B2. Robert Miguel, CFA, is a portfolio manager for a large investment advisory firm. In appreciation of his impressive portfolio returns last quarter, one of his clients, Kevin Goodman, has invited Miguel and his wife to be his guests at his luxury suite for a major league baseball playoff game. Miguel, a baseball fan, accepts the invitation and attends the game. The next day at work, Miguel doesn't mention to his supervisor that he attended the game as a guest of the client. According to the Standard concerning independence and objectivity and the Standard on knowledge of the law, Miguel's actions are in violation of."A. both of these Standards.B. neither of these Standards.C. only one of these Standards.【答案】A3. Ann Smith, CFA, calls Bill Jones, CFA, and tells him that her research shows that Biokem Company is underpriced and that earnings per share will exceed $3.00 this year. Jones had never heard of Biokem before her call but knows that Smith is widely considered to be the best analyst in her sector. Smith's research has been released publicly, and Smith tells Jones he's "welcome to it." After their conversation, Jones arranges a conference call with his firm's portfolio managers and announces that Biokem is underpriced and will likely earn over $3.00 per share this year. During the call with the portfolio managers, Jones does not reference his conversation with Smith. According to the Standards that concern misrepresentation and diligence and reasonable basis, Jones violated:A. both of these Standards.B. neither of these Standards.C. only one of these Standards.【答案】A4. Doug Watson, CFA, serves in a sales position at Sommerset Brokerage, a registered investment adviser. As part of his employment,he is expected to entertain clients. Frequently at these client outings, Watson drinks excessively. On one occasion, after dropping off a client, Watson was cited by local police for misdemeanor public intoxication. According to the Standard on knowledge of the law and the Standard on misconduct, Watson is in violation of:A. both of these Standards.B. neither of these Standards.C. only one of these Standards.【答案】B5. Kevin Richards is a performance analyst for Reliable Advisors, a retail investment advisory and consulting firm. Richards, who is a Level I CFA candidate, was hired-as part of the firm's efforts to attract CFA candidates into critical areas of the firm, such as performance measurement and attribution. Richards' supervisor instructs him to reference the firm's compliance with GIPS in marketing materials to attract more clients. For Richards' reference to the firm's GIPS compliance to be accurate, Reliable is least likely required to:A. apply GIPS compliance firm wide and not only to the specific asset classes mentioned in the marketing materials.B. claim compliance with GIPS only if it has a compliant performance history of five years or more.C. include all discretionary fee-paying accounts in composites basedon their investment objectives and/or strategies.【答案】B6. Peter Taylor, a CFA charterholder and a food industry analyst for a large investment firm, has been invited by Sweet Pineapple Co. to visit the firm's processing plants in Hawaii. The Standard concerning independence and objectivity recommends that Taylor:A. use and pay for commercial transportation, if available.B. obtain written permission from his employer before he accepts this invitation.C. decline this invitation if he issues recommendations on the firm's securities.【答案】A7. Ruth Brett, a Level I CFA candidate, is a research analyst for a large investment firm. Due to a demanding work schedule with long hours, Brett was unable to attend any sort of exam preparation class, and has only been able to study sporadically. Feeling nervous and unprepared the night before the exam, Brett writes a few key notes on the bottom of her shoe. At the exam, Brett sees the large number of proctors present and decides not to risk getting caught and does not look at her shoe. According to the CFA Institute Code of Ethics and Standards of Professional Conduct, Brett is:A. not in violation of any Standard or the Code of Ethics because shedid not use the notes.B. in violation of the Code of Ethics for bringing the notes into the examination room but is not in violation of any Standard because she did not use the notes.C. in violation of both the Code of Ethics and the Standard governing conduct as Members and Candidates in the CFA Program for taking the notes into the examination room.【答案】C8. Which of the following is least likely included in the CFA Code of Ethics? Members of CFA Institute must:A. place their clients' interests before their employer's interests.B. strive to maintain and improve the competence of others in the profession.C. use reasonable care and exercise independent professional judgment.【答案】A9. In formulating her report on GammaCorp's common stock, Barb Kramer, CFA, did a complex series of statistical tests on the company's past sales and earnings. Based on this statistical study, Kramer stated in her report that, "GammaCorp's earnings growth for the next five years will average 15% per year." Her conclusion was based in part on a regression analysis with a high level of statistical significance. Has Kramerviolated the Standard on communication with clients and prospective clients?A. Yes, because she didn't give complete details of the statistical model used.B. Yes, because she failed to indicate that 15% growth is an estimate.C. No, because her projections are within the generally accepted bounds of statistical accuracy.【答案】B10. Alpha Advisors Inc. is an investment management firm with a client base that ranges from individuals to large foundations. Which of the following firm policies is least appropriate if Alpha adopts the Code and Standards? Alpha:A. monitors the personal trading activity of firm personnel and requires them to pre-clear personal trades.B. regularly calls larger accounts first after changes in investment recommendations have been faxed to all clients.C. excludes client accounts of family members of employees from participating in IPOs.【答案】C11. Dudley Thompson is a bond salesman for a small broker/dealer in London. His firm is the lead underwriter on a new junk bond issue forthe Ibex Corporation. In order to stimulate sales of the new issue, Thompson calls all of his accounts over £1,000,000, many of which belong to elderly clients with low risk tolerances, and tells them that the Ibex issue is a fantastic opportunity for high returns that is not to be missed. Thompson also posts overly optimistic projections for Ibex's performance on several Internet "chat rooms" in order to increase the price of the bond issue and enhance his clients' returns on the investment. According to the Standards concerning market manipulation and suitability, Watson is in violation of:A. both of these Standards.B. neither of these Standards.C. only one of these Standards.【答案】A12. Rob Elliott, a CFA candidate, is an analyst with a large asset management firm. His personal portfolio includes a large amount of common stock of TECH Inc., a semiconductor company which his firm does not currently follow. The director of the research department has just asked Elliott to analyze TECH and write a report about its investment potential, which will be distributed to clients and prospective clients. Based on the CFA Institute Standards of Professional Conduct, the most appropriate course of action for Elliot to take would be to:A. sell his shares of TECH before completing the report.B. decline to write the report.C. disclose the ownership of the stock to his employer and in the report if he writes it.【答案】C13. Antonio Mendoza, CFA, is an investment manager in private practice under the name Mendoza Investments. Mendoza solicits new business by making brief presentations describing his investment performance record. At each presentation, he makes available a 1-page information sheet that outlines his performance history for the past ten years. His telephone number is on the sheet for prospective clients who wish to contact him for additional supporting information. At the bottom of the sheet the following is stated: "Mendoza Investments has prepared and presented this report in compliance with the Global Investment Performance Standards (GIPS?).'' Mendoza's information sheet:A. does not comply with GIPS.B. violates the Standard that concerns performance presentation.C. complies with both GIPS and the Standard that concerns performance presentation.【答案】C14. Anne Franklin, CFA, is an analyst who covers technology stocks for Medallion Investments. Franklin frequently meets with company management and makes site visits to company facilities. Cynthia Lucas,chief technology officer for Level Tech, tells several analysts, including Franklin, during a conference call that overseas shipments of the company's important new product are going to be delayed due to manufacturing defects, which she expects are correctable. This information has not been released in any other format. Medallion manages discretionary accounts for Lucas and Franklin. Subsequent to her meeting, Franklin sends a note to Medallion's investment personnel telling them to "sell the stock in all client accounts," as the shipment information is significant and contrary to recent earnings guidance from the company. Franklin's use of the information received from Lucas:A. violates the Code and Standards, as Franklin received material nonpublic information that should not have been acted upon.B. violates the Code and Standards, since she directed the sale in all client accounts instead of acting in the best interests of her own clients.C. does not violate the Code and Standards, as Franklin was adhering to her fiduciary duty to Medallion's clients by sharing the information and recommending a "Sell."【答案】A15. R.J. Young is meeting with a new client for the first time. At this meeting, he will be gathering information about the client in order to assess the client's investment objectives and constraints. As he does for all of his clients, Young will then prepare a written investment policystatement (IPS). According to the recommended procedures for complying with the Standard on suitability, which of the following statements regarding an IPS is least accurate?A. An IPS should describe the roles and responsibilities of the adviser and client.B. A member or candidate is not responsible for financial information withheld by the client.C. A client's IPS must be updated at least quarterly to reflect any changes in their investment profile.【答案】C16. For years John Berger, a CFA charterholder and CEO of a company, relied upon a set of reasonable procedures for preventing violations of the Code and Standards of Professional Conduct in the firm. To not be liable for a violation of the Standards, Berger must:A. do nothing more than have the set of procedures in place as stated.B. periodically review the procedures.C. both periodically review the procedures and ensure the procedures are monitored and enforced.【答案】C17. According to the Standards of Practice Handbook, in order to fulfill the basic provision and procedure of the suitability standards,should a client’s written investment polity statement (IPS): Take performance measurement Be reviewed at least annually?benchmarks into consideration?A. Yes YesB. Yes NoC. No Yes【答案】A18. An analyst meets with a new client. During the meeting, the analyst sees that the new client’s portfolio is heavily invested in one over-the-counter stock. The analyst has been following the stock and thinks it will perform well in the long run. The analyst arranges through a brokerage firm to simultaneously sell a large number of shares of the stock via a series of cross trades from the new client’s portfolio to various existing clients. He arranges the trades to be executed at a price that approximates the current market price. This action is:A. not in violation of the Standards.B. a violation of Standard III(A), Loyalty, Prudence, and Care.C. a violation of Standard III(B), Fair Dealing.【答案】A19. An analyst gathered the following information about the monthly returns over the same time period from two diversifiedinvestment portfolios:The analyst stated that Portfolio X is negatively skewed and Portfolio Y is positively skewed. Is the analyst correct with respect to: Portfolio X?Portfolio Y?A. No NoB. No YesC. Yes No【答案】A20. A pension fund expects to pay obligations of $5, 000, 000 4 years from today. The fund will fund this obligation by making 5 annual, equal deposits into an account earning 3%, with the first deposit occurring today, and the last occurring at the time the obligation is due. What size must each annual deposit be?A. $904,874.B. $941,773.C. $1,125,662.【答案】B21. An investor plans to retire eight years from today. To maintain her standard of living through retirement, she needs to have $2.5 million accumulated when she retires. Her portfolio is currently valued at $1.2million and is expected to earn 7.0 percent annually. The minimum annual amount she must save at the beginning of each of the next eight years to achieve a retirement accumulation of $2.5 million is closest to:A. $0B. $31,875C. $39,914【答案】C22. An analyst asked an junior associate to evaluate the performance of group of mutual funds over the last 10 years. The associate calculated the following performance statistics:The analyst suspects that the associate has made some errors in calculating the performance statistics. Based only on the expected mathematical relationships between the two measures of return and between the two measures of dispersion calculated by the associate, did the associate most likely make errors in calculating the statistics associated with:Mutual Fund A?Mutual Fund B?A. No YesB. Yes NoC. Yes Yes【答案】C23. When using stock return data, a geometric mean return calculation is most likely preferred over a geometric mean calculation because:A. return data can be negative.B. return data can be less than one.C. the geometric mean return is closer in value to the arithmetic mean.【答案】A24. An analyst is investigating the distribution of the SMG stock’s return over time. He calculated the mode of 12 percent, the mean of 11 percent, and the median of 11.5 percent. The distribution can best be described as:A. positively skewed, with a long tail on the left side.B. negatively skewed, with a long tail on the left side.C. positively skewed, with a long tail en the right side.【答案】B25. An analyst collects the following set of past stock returns: -2.3%, -5.1%, 7.6%, 8.2%, 9.1%, and 9.8%. Which of the following measures of return is most likely the highest?A. Median returnB. Geometric mean returnC. Arithmetic mean return【答案】A26. An investor is considering investing in one of the three following portfolios:If the investor's minimum acceptable return is 5%, the optimal portfolio using Roy's safety-first criterion is:A. Portfolio X.B. Portfolio Y.C. Portfolio Z.【答案】C27. An analyst is testing the hypothesis that the variance of monthly returns for Index L equals the variance of monthly returns for Index M based on samples of 50 monthly observations. The sample variance of Index L returns is 0.085, whereas the sample variance of Index M returns is 0.084. Assuming the samples are independent and the returns are normally distributed, which of the following represents the most appropriate test statistic?A.B.C.【答案】A28. If the probability of event J multiplied by the probability of event K is not equal to the joint probability of events J and K, then events J and K are most likely:A. dependent events.B. independent events.C. mutually exclusive events.【答案】A29. Alan Barnes, CFA, is interested in the expected quarterly return on FTSE 100 stock index. He has data for the last five years and calculates the average return on the index over the last 20 quarters. This average return:A. is different from the statistic he is trying to estimate by the amount of the sampling error.B. overstates the return because he should divide by the square root of 20 when using a mean value.C. overstates the expected return because he should have used the geometric mean and not the simple average.【答案】A30. Which of the following statements regarding the significance level of a hypothesis test is most accurate?A. Given a significance level of 5%, a test will reject a true null hypothesis 5% of the time.B. If the significance level of a test is 5%, it will yield the correct decision about the null hypothesis 95% of the time.C. If the significance level of a test is 95%, it will yield the correct decision about the null hypothesis 95% of the time.【答案】A31. An analysis determined that approximately 99 percent of the observations of daily sales for a company were within the interval from $250,000 to $580,000 and that daily sales for the company were normally distributed. The mean daily sales and standard deviation of daily sales, respectively, for the company were closest to:Mean daily sales Standard deviation of daily salesA. $415,000$41,667B. $415,000$55,000C. $355,115$41,667【答案】B32. The joint probability of returns, for securities A and B, are as follows:The covariance of returns of A and B is:A. 6%B. 12%C. 24%【答案】A33. Does the quantity theory of money assume that in the long run:Potential GDP is influenced the price level is influencedby the quantity of money?by the quantity of money?A. No NoB. No YesC. Yes No【答案】B34. Which of the following condition is least likely to be the obstacle to the efficient allocation of productive resources:A. Price controlsB. Taxes and trade restrictionsC. Perfect competition【答案】C35. Which of the following condition is least likely to be the reason that a black market is generally inefficient:A. Unenforceable contractB. Increased prices required by suppliers due to the risk of prosecutionC. Intensive control by the government【答案】C36. Which of the following describes the difference between perfect competition and monopolistic competition? Firms in monopolistic competition:A. Provide differentiated products.B. Decide on the output where marginal revenue equals marginal cost.C. Are free to enter and exit the market.【答案】A37. Generally speaking, when the Herfindahl-Hirschman Index (HHI) is calculated by summing the squared percentage market shares, how many largest firms’data is used in a industry, supposing there are a large number of firms in the industry:A. 4B. 10C. 50【答案】C38. If accurately calculated, will the amount of accounting profit fora firm generally:Be below that firm’s economic consider both that firm’s profit for the same time Defied?explicit and implicit costs?A. No NoB. No YesC. Yes No【答案】A39. The demand for a given resource will most likely decrease if the prices of its substitute resources and complementary resources, respectively:Prices of substitute resources Price of complementary resourcesA. Increase IncreaseB. Increase DecreaseC. Decrease Increase【答案】C40. With respect to the different views about short-term fluctuations in the economy, what is the primary driver of aggregate demand according to the:Keynesian view?Classical view?A. Future expectations Technological changeB. Future expectations The supply of moneyC. Current price level Technological change【答案】A41. At low wage rates, will an increase in the wage rate most likely result in:The substitution effect?An increase in the supply of labor?A. Yes YesB. Yes NoC. No Yes【答案】A42. An analyst does research about technologically efficient and economically efficient. A method of production is economically efficient when:A. it uses the most up-to-date technology.B. the cost of human capital is as low as possible.C. the cost of producing a given output is as low as possible.【答案】C43. Given the following possibilities, which one results in an increase in total consumer expenditures?A. demand is elastic and price rises.B. demand is inelastic and price falls.C. demand is inelastic and price rises.【答案】C44. Suppose that rubber is the primary input in the production of golf balls. If the price of rubber increases while all else remains constant, then in the short-run:A. The marginal and average variable cost curves shift upward, but not the average total or average fixed cost curves.B. The average total and average variable cost curves shift upward, but the marginal and average fixed cost curves will shift downward.C. The marginal, average variable, and average total cost curves will shift upward, but the average fixed cost curve will not shift.【答案】C45. In which of the following situations when recording inventory ata value greater than its historical cost is permitted?A. The value is a reversal of a pervious write-down.B. The inventory consists of agricultural products.C. Financial statements are prepared using U.S.GAAP.【答案】B31. For an asset with a price of 1000, which of the following price targets is least likely based on Fibonacci ratio?A. 618.B. 1294.C. 1618.【答案】B32. A portfolio of non-dividend-paying common stocks earned a geometric mean return of 5 percent between 1 January 2002 and 31 December 2008. The arithmetic mean return for the same period was 6 percent. If the market value of the portfolio at the beginning of 2002 was $100,000, the market value of the portfolio at the end of 2008 was closest to:A. $137,000B. $140,710C. $141,030【答案】B33. Which of the following is not true regarding to zero economic profit?A. The net present value of cash flows when discounted at the firms cost of capital will equal to zero.B. The firm is just profitable enough to make a normal return.C. The return for the firm is insufficient for the risk taken.【答案】C34. An analyst does research on price discrimination’s impact. Compared to using a single price in a market characterized by monopolistic competition, using price discrimination when two customergroups have different demand elasticities is least likely to result in:A. new productive ventures.B. an increase in total output.C. lower prices for both groups.【答案】C资料来源:文得学习网,更多考试资料题库视频,上文得学习网查找。

cfa一级考试真题及答案详解

cfa一级考试真题及答案详解

cfa一级考试真题及答案详解CFA一级考试真题及答案详解1. 问题:在计算投资组合的预期收益率时,以下哪种资产的预期收益率对投资组合的预期收益率影响最大?A. 资产A,占投资组合的10%B. 资产B,占投资组合的20%C. 资产C,占投资组合的30%D. 资产D,占投资组合的40%答案:D解析:投资组合的预期收益率是通过加权平均各个资产的预期收益率来计算的。

权重越大的资产对投资组合的预期收益率影响越大。

因此,资产D作为权重最大的资产,其预期收益率对投资组合的预期收益率影响最大。

2. 问题:以下哪种金融工具最适合用于对冲利率风险?A. 股票B. 债券C. 利率互换D. 货币期权答案:C解析:利率互换是一种利率衍生品,允许两方交换利率支付流。

这种金融工具可以用来对冲利率风险,因为它允许投资者锁定未来的利率水平,从而减少利率变动对投资组合价值的影响。

3. 问题:在进行财务分析时,以下哪种比率最能反映公司的流动性状况?A. 流动比率B. 速动比率C. 资产负债率D. 权益乘数答案:A解析:流动比率是衡量公司短期偿债能力的指标,它通过比较公司的流动资产和流动负债来计算。

一个较高的流动比率表明公司有足够的流动资产来覆盖其短期负债,因此最能反映公司的流动性状况。

4. 问题:在计算经济增加值(EVA)时,以下哪个因素不是必要的?A. 公司的税后营业利润B. 公司的资本成本C. 公司的总资产D. 公司的权益乘数答案:D解析:经济增加值(EVA)是通过从公司的税后营业利润中减去资本成本来计算的。

资本成本反映了公司资本的机会成本,而权益乘数并不是计算EVA所必需的。

EVA的计算公式为:EVA = NOPAT - (WACC * 资本投入)。

结束语:通过以上对CFA一级考试真题的分析,可以看出,理解各个金融概念和工具的基本原理对于通过考试至关重要。

希望这些真题及答案详解能够帮助考生更好地准备考试,提高通过率。

CFA考试模拟试题库及答案解析

CFA考试模拟试题库及答案解析

CFA考试模拟试题库及答案解析CFA考试是金融行业内非常重要的认证考试之一,通常由金融投资领域的从业人员来参加。

为了帮助考生更好地备考,许多机构和网站提供了CFA考试模拟试题库,并结合详细的答案解析,以帮助考生提高他们的考试准备水平。

CFA考试模拟试题库通常是由一系列精心编写的试题组成,覆盖了考试的各个科目。

这些试题大多数都模拟了真实的CFA考试题型和难度水平,能够帮助考生熟悉考试内容和要求。

试题库中的题目可以按照科目进行分类,并可以按照不同的难度等级进行排序。

此外,一些试题库还提供了答题时间和得分情况的记录,以帮助考生对自己的答题速度和准确性进行评估。

对于每道试题,答案解析是考生重点关注的部分。

答案解析通常会提供每个选项的解释和推理过程,帮助考生理解各个选项的含义和与题目的关系。

更重要的是,答案解析还会解释为什么某个选项是正确答案,为什么其他选项是错误的。

这些解析非常有价值,因为它们可以帮助考生理解CFA考试的考点,提高答题的准确性和速度。

在应对CFA考试模拟试题时,考生可以通过以下步骤来学习和强化自己的知识:1. 熟悉题目类型:CFA考试的题型涵盖了单项选择题、多项选择题、材料分析题等,考生需要了解每个题型的特点和解题技巧。

2. 逐题练习:通过模拟试题库,考生可以从易到难逐步练习不同类型的题目。

在练习过程中,考生要注意分析每个选项的优缺点,思考为什么正确答案是正确的,为什么错误答案是错误的。

3. 查漏补缺:通过模拟试题库的答案解析,考生可以找出自己在答题过程中遇到的错误和问题。

针对这些错误和问题,考生可以查漏补缺,强化自己的知识点。

4. 考点总结:在学习的过程中,考生可以总结每个科目的重点考点,并做好相关知识的梳理和总结。

这样可以帮助考生在备考过程中更加有针对性地学习和复习。

总之,CFA考试模拟试题库及答案解析是CFA考生备考的重要资料之一。

考生可以通过模拟试题的练习和答案解析的学习,提高自己的答题技巧和知识水平,为顺利通过CFA考试做好充分准备。

cfa一级考试模拟试题及答案

cfa一级考试模拟试题及答案

cfa一级考试模拟试题及答案1. 以下哪项不是CFA一级考试中伦理与职业标准科目的主要内容?A. 专业行为准则B. 全球投资绩效标准C. 投资组合管理D. 职业行为标准答案:C2. 在CFA一级考试中,关于货币的时间价值,以下哪个公式是不正确的?A. 未来价值(FV)= 现值(PV)× (1 + r)^nB. 现值(PV)= 未来价值(FV)/ (1 + r)^nC. 未来价值(FV)= 现值(PV)× (1 - r)^nD. 现值(PV)= 未来价值(FV) × (1 - r)^n答案:C3. CFA一级考试中,关于投资组合理论,以下哪项是正确的?A. 投资组合的风险随着组合中资产数量的增加而增加B. 投资组合的风险随着组合中资产数量的增加而减少C. 投资组合的风险与组合中资产数量无关D. 投资组合的风险总是高于组合中任一资产的风险答案:B4. 在CFA一级考试中,关于固定收益证券,以下哪项说法是错误的?A. 债券的收益率与价格呈反比关系B. 长期债券对利率变化的敏感性高于短期债券C. 债券的票息率越高,其价格对利率变化的敏感性越高D. 债券的到期时间越长,其价格对利率变化的敏感性越高答案:C5. CFA一级考试中,关于衍生品,以下哪项是正确的?A. 期权的时间价值随着到期时间的增加而减少B. 期权的内在价值等于执行价格与标的资产价格之差C. 期权的执行价格与标的资产价格之差称为期权的时间价值D. 期权的时间价值随着到期时间的减少而增加答案:D结束语:以上是CFA一级考试的部分模拟试题及答案,希望对您的备考有所帮助。

请继续努力学习,祝您在考试中取得优异成绩。

2024年cfa一级题库

2024年cfa一级题库

2024年cfa一级题库
道德与专业标准
CFA协会道德及行为准则概述,包括其原则、准则和定例道德困境识别、分析和解决的框架
利益冲突、信息保护和公平交易的原则
专业技能和责任的标准
数量方法
数据摘要、展示和解释
概率论的基础知识,包括事件、条件概率和贝叶斯定理
假设检验,包括t检验、Z检验、方差分析和卡方检验
回归分析,包括一元线性回归和多元回归
时间序列分析,包括平稳性检验和指数平滑
经济学
微观经济学的核心原理,包括供求、消费者行为和企业行为
宏观经济学的核心原理,包括国民收入、通货膨胀和失业
货币和财政政策的作用
国际经济学,包括汇率和国际贸易
经济增长和发展理论
财务报表分析
财务报表的三个主要组成部分:资产负债表、利润表和现金流量表
财务报表分析的定性和定量方法
应收账款、存货和固定资产的分析
负债和所有者权益的分析
损益表分析
公司金融
资本预算技术,包括净现值、内含报酬率和收益指数
资本结构和融资决策
股息政策
风险和收益率的关系
权益投资
股票估值模型,包括贴现现金流模型、股利贴现模型和市盈率债券估值和利率风险
投资组合管理,包括多元化、资产配置和绩效评估衍生工具
期货、期权和互换合约的基本概念
衍生工具的定价和使用
衍生工具的风险管理
其他相关主题
投资分析师的角色和责任
金融业监管
职业道德和行为守则的实际应用。

特许金融分析师(CFA)测试题与答案

特许金融分析师(CFA)测试题与答案

特许金融分析师(CFA)测试题与答案1.以下理解或做法符合金融理财定义的是( )。

A.赵先生认为金融理财是有钱人考虑的事情,自己收入微薄不需要金融理财服务B.贺先生认为金融理财的目的是为了追求每个生命周期收入均大于支出C.理财师小王每遇客户就推荐所在机构新推出的理财产品D.理财师小张分析所在机构新推出的理财产品很符合客户吴先生的需求,于是向吴先生推荐该产品正确答案:D2.根据FPSBChina的规定,完成AFP或CFP资格认证后,每( )必须再认证一次。

A.2年B.3年C.4年D.5年正确答案:A3.根据FPSBChina对金融理财的定义,下列说法错误的是( )。

A.金融理财不是产品推销,是一种综合金融服务B.金融理财强调由专业理财人员为客户提供综合财务规划C.金融理财强调由专业理财人员通过规范的操作流程,帮助客户实现人生各阶段的目标D.金融理财的目标是帮助客户迅速致富正确答案:D解析:根据FPSBChina对金融理财的定义,金融理财的目标是追求一生财务资源收支的平衡、追求支出的合理控制、财务风险的控制及进行财富管理。

帮助客户迅速致富不是金融理财的目标,因此D是错误的。

4.根据《金融理财师资格认证办法》,2010年12月31日,下列情形符合AFP资格认证工作经验时间要求的是( )。

A.张某1997年大专毕业,在律师事务所为客户做离婚和遗产筹划,2001年初开始一直从事行政管理工作B.王某2008年本科毕业,一直在银行从事文秘工作C.李某2009年硕士研究生毕业,一直在保险公司从事客户寿险的规划工作D.赵某2010年博士研究生毕业,一直在证券公司从事人力资源管理工作正确答案:C解析:根据《金融理财师资格认证办法》,AFP资格认证对于工作经验的要求包括:必须在金融理财相关行业从事金融理财的相关工作,研究生需相关工作经验1年,本科2年,大专3年,工作经验的有效期为最近10年。

A中张某虽然从事理财的相关工作,也达到了大专3年的标准,但是其1997~2000年的工作经验已经超出了认定的有效期;B、D中的王某和赵某虽然在金融机构工作,但从事的均不是与金融理财相关的工作。

历年CFA考试真题及答案解析

历年CFA考试真题及答案解析

历年CFA考试真题及答案解析1、The nominal (quoted) annual interest rate on an automobile loan is 10%. The effective annual rate of the loan is 10.47%. The frequency of compounding periods per year for the loan is closest to:【单选题】A.weekly.B.monthly.C.quarterly.正确答案:B答案解析:“The Time Value of Money,” Richard A. DeFusco, CFA, Dennis W. McLeavey, CFA, Jerald E. Pinto, CFA, and David E. Runkle, CFA2013 Modular Level I, Vol. 1, Reading 5, Section 3.3Study Session 2–5–c, dCalculate and interpret the effective annual rate, given the stated annual interest rate and the frequency of compounding. Solve time value of money problems for different frequencies of compounding:B is correct. Use the formula for effective annual rate:Iteratively substitute the possible frequency of compounding until the EAR is 10.47%.Thus, the correct answer is monthly compounding.2、Which of the following is a constraint as defined in the International Financial Reporting Standards (IFRS) Framework for the Preparation and Presentation of Financial Statements?【单选题】A.NeutralityB.TimelinessC.Going concern正确答案:B答案解析:“Financial Reporting Standards,” Thomas R. Robinson, CFA, Jan Hendrik van Greuning, CFA, Karen O’Connor Rubsam, CFA, R. Elaine Henry, CFA, and Michael A. Broihahn, CFATimeliness is a constraint in the IFRS Framework. Neutrality is a factor that contributes to reliability and going concern is an assumption of the Framework.3、Which method of calculating the firm’s cost of equity is most likely to incorporate the long-run return relationship between the firm's stock and the market portfolio?【单选题】A.Dividend discount modelB.Capital asset pricing modelC.Bond-yield-plus risk-premium正确答案:B答案解析:“Cost of Capital” Yves Courtois, CFA, Gene C. Lai, and Pamela Peterson Drake, CFAThe capital asset pricing model uses the firm’s equity beta, which is computed from a market model regression of the company's stock returns against market returns.4、For a 90-day U.S. Treasury bill selling at a discount, which of the following methods most likely results in the highest yield?【单选题】A.Money market yieldB.Discount-basis yieldC.Bond equivalent yield正确答案:C答案解析:“Working Capital Management,” Edgar Norton, Jr., Kenneth L. Parkinson, and Pamela Peterson Drake5、An investor gathers the following data.To estimate the stock's justified forward P/E, the investor prefers to use the compounded annual earnings growth and the average of the payout ratios over the relevant period (i.e., 2008–2011). If the investor uses 11.5% as her required rate of return, the stock's justified forward P/E is closest to:【单选题】A.10.B.12.C.21.正确答案:C答案解析:“Equity Valuation: Concepts and Basic Tools,” John J. Nagorniak and Stephen E.Wilcox6、A bond portfolio manager is considering three Bonds – A, B, and C – for his portfolio. Bond A allows the issuer to call the bond before stated maturity, Bond B allows the investor to put the bond back to the issuer before stated maturity, and Bond C contains no embedded options. The bonds are otherwise identical. The manager tells his assistant, “Bond A and Bond B should have larger nominal yield spreads to a U.S. Treasury than Bond C to compensate for their embedded options.”Is the manager most likely correct?【单选题】A.Yes.B.No, Bond A’s nominal yield spread should be less than Bond C’s.C.No, Bond B’s nominal yield spread should be less than Bond C’s.正确答案:C答案解析:“Understanding Yield Spreads,” Frank J. Fabozzi, CFAC is correct because Bond B’s embedded put option benefits the investor and the yield spread willtherefore be less than the yield spread of Bond C, which does not contain this benefit.7、Which of the following characteristics is best described that the information in financialstatements can influence user's economic decisions or affect user's evaluationof past events or forecasts of future events in accordance with the IFRSframework's definitions and recognition criteria?【单选题】A.Relevance.parability.C.Faithful representation.正确答案:A答案解析:根据IFRS的条款,财务报表的两个基本特性使得这些财务信息有用,这两个特性包括相关性(relevance)和公允陈述(faithful representation)。

楷博财经cfa期末考试题目及答案

楷博财经cfa期末考试题目及答案

楷博财经cfa期末考试题目及答案一、单项选择题(每题2分,共20分)1. CFA考试的全称是什么?A. Certified Financial AnalystB. Chartered Financial AnalystC. Certified Financial AccountantD. Certified Financial Advisor答案:B2. 以下哪个不是CFA考试的科目?A. 伦理与职业标准B. 数量分析C. 经济学D. 管理学答案:D3. CFA考试共分为几个级别?A. 1B. 2C. 3D. 4答案:C4. CFA考试的通过标准是什么?A. 60%B. 70%C. 80%D. 90%5. CFA考试的合格成绩有效期是多久?A. 1年B. 2年C. 3年D. 5年答案:C6. CFA考试的报名费用是多少?A. 1000美元B. 2000美元C. 3000美元D. 4000美元答案:B7. CFA考试的考试形式是什么?A. 笔试B. 机考C. 面试D. 口试答案:B8. CFA考试的考试时间是多久?A. 3小时B. 4小时C. 5小时D. 6小时答案:C9. CFA考试的考试语言是什么?B. 法语C. 德语D. 西班牙语答案:A10. CFA考试的考试地点通常在哪里?A. 学校B. 考试中心C. 公司D. 家里答案:B二、多项选择题(每题3分,共15分)1. CFA考试的考试科目包括以下哪些?A. 投资工具B. 资产评估C. 投资组合管理D. 风险管理答案:ABC2. CFA考试的报名条件包括以下哪些?A. 拥有学士学位B. 拥有专业工作经验C. 通过CFA一级考试D. 通过CFA二级考试答案:AB3. CFA考试的考试题型包括以下哪些?A. 单选题B. 多选题D. 论述题答案:AB4. CFA考试的考试内容涉及以下哪些领域?A. 公司金融B. 固定收益C. 衍生品D. 行为金融学答案:ABCD5. CFA考试的考试通过后可以获得以下哪些资格?A. CFA一级证书B. CFA二级证书C. CFA三级证书D. CFA特许金融分析师答案:D三、简答题(每题5分,共20分)1. 简述CFA考试的三个级别分别考察哪些内容?答案:CFA一级主要考察投资工具、资产评估和投资组合管理的基础知识;CFA二级主要考察资产评估的深入分析和投资组合管理的应用;CFA三级则侧重于投资组合管理的实践和案例分析。

CFA考试试题汇编(含答案)

CFA考试试题汇编(含答案)

CFA考试试题汇编1、The nominal (quoted) annual interest rate on an automobile loan is 10%. The effective annual rate of the loan is 10.47%. The frequency of compounding periods per year for the loan is closest to:【单选题】A.weekly.B.monthly.C.quarterly.正确答案:B点拨::“The Time Value of Money,” Richard A. DeFusco, CFA, Dennis W. McLeavey, CFA, Jerald E. Pinto, CFA, and David E. Runkle, CFA2013 Modular Level I, Vol. 1, Reading 5, Section 3.3Study Session 2–5–c, dCalculate and interpret the effective annual rate, given the stated annual interest rate and the frequency of compounding. Solve time value of money problems for different frequencies of compounding:B is correct. Use the formula for effective annual rate:Iteratively substitute the possible frequency of compounding until the EAR is 10.47%.Thus, the correct answer is monthly compounding.2、Which of the following is a constraint as defined in the International Financial Reporting Standards (IFRS) Framework for the Preparation and Presentation of Financial Statements?【单选题】A.NeutralityB.TimelinessC.Going concern正确答案:B点拨::“Financial Reporting Standards,” Thomas R. Robinson, CFA, Jan Hendrik van Greuning, CFA, Karen O’Connor Rubsam, CFA, R. Elaine Henry, CFA, and Michael A. Broihahn, CFATimeliness is a constraint in the IFRS Framework. Neutrality is a factor that contributes to reliability and going concern is an assumption of the Framework.3、Which method of calculating the firm’s cost of equity is most likely to incorporate the long-run return relationship between the firm's stock and the market portfolio?【单选题】A.Dividend discount modelB.Capital asset pricing modelC.Bond-yield-plus risk-premium正确答案:B点拨::“Cost of Capital” Yves Courtois, CFA, Gene C. Lai, and Pamela Peterson Drake, CFAThe capital asset pricing model uses the firm’s equity beta, which is computed from a market model regression of the company's stock returns against market returns.4、For a 90-day U.S. Treasury bill selling at a discount, which of the following methods most likely results in the highest yield?【单选题】A.Money market yieldB.Discount-basis yieldC.Bond equivalent yield正确答案:C点拨::“Working Capital Management,” Edgar Norton, Jr., Kenneth L. Parkinson, and Pamela Peterson Drake5、An investor gathers the following data.To estimate the stock's justified forward P/E, the investor prefers to use the compounded annual earnings growth and the average of the payout ratios over the relevant period (i.e., 2008–2011). If the investor uses 11.5% as her required rate of return, the stock's justified forward P/E is closest to:【单选题】A.10.B.12.C.21.正确答案:C点拨::“Equity Valuation: Concepts and Basic Tools,” John J. Nagorniak and Stephen E.Wilcox6、A bond portfolio manager is considering three Bonds – A, B, and C – for his portfolio. Bond A allows the issuer to call the bond before stated maturity, Bond B allows the investor to put the bond back to the issuer before stated maturity, and Bond C contains no embedded options. The bonds are otherwise identical. The manager tells his assistant, “Bond A and Bond B should have larger nominal yield spreads to a U.S. Treasury than Bond C to compensate for their embedded options.”Is the manager most likely correct?【单选题】A.Yes.B.No, Bond A’s nominal yield spread should be less than Bond C’s.C.No, Bond B’s nominal yield spread should be less than Bond C’s.正确答案:C点拨::“Understanding Yield Spreads,” Frank J. Fabozzi, CFAC is correct because Bond B’s embedded put option benefits the investor and the yield spread willtherefore be less than the yield spread of Bond C, which does not contain this benefit.7、Which of the following characteristics is best described that the information in financialstatements can influence user's economic decisions or affect user's evaluationof past events or forecasts of future events in accordance with the IFRSframework's definitions and recognition criteria?【单选题】A.Relevance.parability.C.Faithful representation.正确答案:A点拨::根据IFRS的条款,财务报表的两个基本特性使得这些财务信息有用,这两个特性包括相关性(relevance)和公允陈述(faithful representation)。

cfa三级道德 简答题

cfa三级道德 简答题

CFA三级考试中的道德(Ethics)部分是一个重要的组成部分,它测试考生对专业道德标准(Standards of Professional Conduct)的理解和应用。

CFA三级考试的道德部分通常包括以下几个方面:
1. 专业责任(Professional Responsibilities)
-要求考生了解和遵守CFA协会的职业行为准则。

-考察考生在面对职业道德困境时如何作出决策。

2. 候选人的职业行为(Candidate Conduct)
-评估候选人个人的职业行为,是否符合CFA协会的期望。

-考察候选人如何处理利益冲突、保密信息和社交媒体的使用。

3. 合规性和监督(Compliance and Supervision)
-了解候选人在监督下的职责,以及如何确保合规性。

-考察候选人如何处理不当行为和违规行为。

4. 投资决策(Investment Decision Making)
-要求考生理解在做出投资决策时,应如何考虑道德和合规性因素。

-考察考生如何将道德标准应用于投资分析、建议和行动中。

简答题通常会要求考生回答关于道德标准的特定情况或问题,考生需要根据CFA协会的职业行为准则,分析问题并给出合理的答案。

这些题目可能会涉及复杂的道德困境,需要考生具备批判性思维和应用能力。

cfa二级考试题目

cfa二级考试题目

选择题关于有效市场假说,以下哪个描述是正确的?A. 在有效市场中,投资者可以通过分析信息来获得超额收益。

B. 有效市场假说认为市场总是错误的,因此投资者应忽视市场信息。

C. 有效市场假说认为市场价格反映了所有可用信息,因此无法通过分析获得超额收益。

(正确答案)D. 有效市场仅适用于股票市场,不适用于债券市场。

在评估公司的资本结构时,以下哪个因素不是考虑的重点?A. 公司的债务与股权比例。

B. 公司的盈利能力。

C. 公司的行业特点。

(正确答案)D. 公司的现金流稳定性。

以下哪个是衡量公司运营效率常用的财务指标?A. 市盈率B. 存货周转率(正确答案)C. 资本充足率D. 杠杆比率在进行股票估值时,以下哪个模型是基于公司未来现金流的贴现值?A. 市盈率模型B. 市净率模型C. 贴现现金流模型(正确答案)D. 相对估值模型关于固定收益证券,以下哪个描述是错误的?A. 固定收益证券的收益率与市场利率变动呈反向关系。

B. 固定收益证券的信用风险通常低于股票。

C. 固定收益证券的价格波动通常小于股票。

(正确答案)D. 固定收益证券的到期收益率考虑了利息再投资的风险。

在投资组合管理中,以下哪个策略旨在通过分散投资来降低风险?A. 市场时机选择策略B. 资产配置策略(正确答案)C. 股票选择策略D. 集中投资策略以下哪个不是衡量投资组合风险的常用指标?A. 标准差B. β系数C. 夏普比率D. 跟踪误差(正确答案)关于期权定价,以下哪个模型是最常用的?A. 费雪方程式B. 黑-斯科尔斯模型(正确答案)C. 有效市场假说D. 卡普兰-米尔斯模型在进行宏观经济分析时,以下哪个指标不是常用的先行指标?A. 制造业订单B. 失业率(正确答案)C. 建筑许可D. 消费者预期指数。

cfa一级考试真题及答案解析

cfa一级考试真题及答案解析

cfa一级考试真题及答案解析CFA一级考试真题及答案解析1. 问题:某投资者持有一个投资组合,其中包含股票A和股票B,股票A的预期收益率为10%,股票B的预期收益率为15%,股票A在投资组合中的权重为60%,股票B的权重为40%。

请问该投资组合的预期收益率是多少?解析:投资组合的预期收益率可以通过加权平均的方式来计算。

具体计算公式为:投资组合预期收益率 = (股票A的预期收益率 * 股票A的权重) + (股票B的预期收益率 * 股票B的权重)。

将题目中给出的数据代入公式,得到投资组合的预期收益率 = (10% * 60%) + (15% * 40%) = 6% + 6% = 12%。

答案:投资组合的预期收益率为12%。

2. 问题:某公司在年初发行了面值为1000元,期限为5年,年利率为5%的债券。

如果市场利率为6%,请问该债券的当前市场价格是多少?解析:债券的市场价格可以通过现值公式来计算,即市场价格 = 债券面值 / (1 + 市场利率)^债券期限。

将题目中给出的数据代入公式,得到债券的市场价格 = 1000 / (1 + 6%)^5 ≈ 735.47元。

答案:该债券的当前市场价格约为735.47元。

3. 问题:某投资者购买了一份看涨期权,执行价格为100元,期权费为5元,到期日时标的资产的市场价格为120元。

请问该投资者的净收益是多少?解析:看涨期权的净收益可以通过以下公式计算:净收益 = (标的资产的市场价格 - 执行价格 - 期权费)。

将题目中给出的数据代入公式,得到净收益 = (120 - 100 - 5) = 15元。

答案:该投资者的净收益为15元。

4. 问题:某公司上一年的营业收入为500万元,营业成本为300万元,销售和管理费用为100万元,财务费用为20万元,投资收益为30万元。

请问该公司上一年的净利润是多少?解析:净利润的计算公式为:净利润 = 营业收入 - 营业成本 - 销售和管理费用 - 财务费用 + 投资收益。

2019年CFA一级notes整理数量分析答案

2019年CFA一级notes整理数量分析答案

数量分析答案Module Quiz 6.11.C Interest rates can be interpreted as required rates of return,discount rates,or opportunity costs of current consumption.A risk premium can be,but is not always,a component of an interest rate.(LOS 6.a, 6.b)2.A Real interest rates are those that have been adjusted for inflation.(LOS 6.b)3.C EAR=[(1+(0.18/12)]12−1=19.56%(LOS6.c)4.C N=30×12=360;I/Y=9/12=0.75;PV=–150,000(1−0.2)=–120,000;FV=0;CPT→PMT=$965.55(LOS6.d)5.C N=1×365=365;I/Y=12/365=0.0328767;PMT= 0;PV=–5,000;CPT→FV=$5,637.37(LOS 6.d)Module Quiz 6.21.C N=15;I/Y=9;PV=–1,000;PMT=0;CPT→FV= $3,642.48(LOS 6.e)2.A N=7;I/Y=8;FV=–10,000;PMT=0;CPT→PV= $5,834.90(LOS 6.e)3.A N=20;I/Y=10;PMT=–50,000;FV=0;CPT→PV= $425,678.19(LOS6.e)4.C This is an annuity due.Switch to BGN mode:N=15; PMT=–8,000;I/Y=11;FV=0;CPT→PV=63,854.92. Switch back to END mode.(LOS 6.e)5.B The key to this problem is to recognize that it is a 4-year annuity due,so switch to BGN mode:N=4;PMT=–1,000;PV=0;I/Y=12;CPT→FV=5,352.84.Switch back to END mode.(LOS6.e)6.A9/0.11=$81.82(LOS 6.e)Module Quiz 6.31.B N=8;PV=–3;FV= 4.50;PMT=0;CPT→I/Y= 5.1989(LOS 6.e)2.C PV=–5,000;I/Y=12;FV=10,000;PMT=0;CPT→N = 6.12.Note to HP12C users:One known problem with the HP12C is that it does not have the capability to round.In this particular question,you will come up with7,although the correct answer is 6.1163.CFA Institute is aware of this problem,and hopefully you will not be faced with a situation on exam day where the incorrect solution from the HP is one of the answer choices.(LOS 6.e)3.B Add up the present values of each single cash flow. PV1=N=1;FV=–500;I/Y=12;CPT→PV=446.43PV2=N=2;FV=–200;I/Y=12;CPT→PV=159.44PV3=N=3;FV=–800;I/Y=12;CPT→PV=569.42 Hence,446.43+159.44+569.42=$1,175.29.(LOS 6.e)4.B PV=–10,000;I/Y=9.5;N=5;FV=0;CPT→PMT= $2,604.36(LOS 6.e)5.B N=10;I/Y=11;FV=–60,000;PV=0;CPT→PMT= $3,588.08(LOS6.e)6.B Two steps:(1)Find the PV of the10-year annuity:N =10;I/Y=9;PMT=–4,000;FV=0;CPT→PV=25,670.63.This is the present value as of the end of Year4;(2)Discount PV of the annuity back four years:N=4;PMT=0;FV=–25,670.63;I/Y=9;CPT→PV=18,185.72.(LOS6.e) Module Quiz7.11.A NPV=4/ 1.10+3/(1.10)2+4/(1.10)3−$10 =–$0.879038million,or–$879,038Calculator approach:CF0=–10;CF1=4;CF2=3;CF3 =4;I=10;NPV=–$0.879038(million)(LOS7.a)2.A NPV=PV(cash inflows)−CF0=($1.8million/0.105)−$15million=$2,142,857.Accept the project.(LOS7.a)3.C As a perpetuity,the following relationship applies: $1.8million/IRR=$15million.Thus,IRR= 1.8/15= 12%.Since IRR>cost of capital(hurdle rate),accept the project.(LOS7.a)4.B If the NPV and IRR methods give conflicting decisions when selecting among mutually exclusive projects,always select the project with the greatest positiveNPV.(LOS7.b)5.C A project will have a negative NPV if its IRR is less than the firm’s cost of capital.(LOS7.b)6.C Both projects should be accepted since both projects have positive NPVs andwill thus increase shareholder wealth.(LOS7.b)7.C(LOS7.c)Module Quiz7.21.C The money-weighted rate of return is the IRR of aninvestment’s net cashflows.(LOS7.d)2.C On your financial calculator,solve for IRR: Calculating Money-Weighted Return With the TI Business Analyst II Plus®(LOS7.d)3.C HPR1=(65+2)/50−1=34%,HPR2=(140+4)/ 130−1=10.77%Time-weighted return=[(1.34)(1.1077)]0.5−1=21.83% (LOS7.d)4.B(1,000/100,000)×(360/95)= 3.79%(LOS7.e)5.A(100,000−99,000)/99,000= 1.01%(LOS7.e)6.B(1+0.0101)365/95−1= 3.94%(LOS7.e)7.B(360×0.0379)/[360–(95×0.0379)]= 3.83%,or (1,000/99,000)(360/95)= 3.83%(LOS7.e)8.C This is actually the definition of the holding period yield.The other answers aretrue statements regarding the bank discount yield.(LOS7.e)8.CModule Quiz8.11.B Poker hand rankings are an ordinal scale.We can saya pair outranks a highcard(therefore,it is not a nominal scale),but we cannot say a pair is some amountgreater than a high card(therefore,it is not an interval scale).(LOS8.a)2.C Intervals within a frequency distribution should always be nonoverlapping andclosed ended so that each data value can be placed into only one interval.Intervalshave no set width and should be set at a width so that data is adequatelysummarized without losing valuable characteristics.(LOS 8.b)3.B An interval is the set of return values that an observation falls within.Simplycount the return intervals on the table—there are five of them.(LOS8.c)4.C The sample size is the sum of all of the frequencies in the distribution,or3+7+3+2+1=16.(LOS8.c)5.C The relative frequency is found by dividing thefrequency of the interval by thetotal number of frequencies.(LOS8.c)6.A In a histogram,the intervals are on the horizontal axis and the frequency is onthe vertical axis.(LOS8.d)Module Quiz8.21.A[22%+5%+–7%+11%+2%+11%]/6=7.3%(LOS8.e)2.B To find the median,rank the returns in order and take the middle value:–7%,2%,5%,11%,11%,22%.In this case,because there is an even number of observations,the median is the average of the two middle values,or(5%+11%)/2=8.0%.(LOS8.e)3.C The mode is the value that appears most often,or11%. (LOS8.e)4.C The range is calculated by taking the highest value minus the lowest value.22%−(–7%)=29.0%(LOS8.g)5.B The mean absolute deviation is found by taking the mean of the absolute valuesof the deviations from the mean.(|22−7.3|+|5−7.3|+|–7−7.3|+|11−7.3|+|2−7.3|+|11−7.3|)/6=7.33%(LOS8.g)6.C The population variance,σ2,is found by taking the mean of all squared deviations from the mean.σ2=[(22−7.3)2+(5−7.3)2+(–7−7.3)2+(11−7.3)2+(2−7.3)2+(11−7.3)2]/6=80.2%2(LOS8.g)7.B The population standard deviation,σ,is found by taking the square root of the population variance.σ==(80.2%2)0.5=8.96%(LOS8.g)8.C The sample variance,s2,uses n–1in the denominator.s2=[(22−7.3)2+(5−7.3)2+(–7−7.3)2+(11−7.3) 2+(2−7.3)2+(11−7.3)2]/(6−1)=96.3%2(LOS8.g)9.A The sample standard deviation,s,is the square root of the sample variance.σ==(96.3)0.5=9.8%(LOS8.g)10.A(15%+19%–8%+14%)/4=10%(LOS8.e)11.A(1.15×1.19×0.92×1.14)0.25−1=9.45%(LOS8.e) PROFESSOR’S NOTEThis question could have been answered very quickly since the geometric mean must be less than the arithmetic mean computed in the preceding problem.12.B(LOS8.e)13.C With eight observations,the location of the65th percentile is:(8+1)×65/100= 5.85observationsThe fifth observation is7and the sixth observation is9, so the value at 5.85observations is7+0.85(9–7)=8.7. (LOS8.f)Module Quiz8.31.C Applying Chebyshev’s inequality,1−[1/(2.5)2]=0.84,or84%.(LOS8.h)2.C The portfolio’s Sharpe ratio is(6.0%–1.0%)/ 2.5% = 2.0.(LOS8.i)3.B A distribution that has a greater percentage of small deviations from the meanand a greater percentage of extremely large deviationsfrom the mean will beleptokurtic and will exhibit excess kurtosis(positive). The distribution will bemore peaked and have fatter tails than a normal distribution.(LOS8.j)4.A A distribution with a mean greater than its median is positively skewed,orskewed to the right.The skew“pulls”the mean.Note: Kurtosis deals with theoverall shape of a distribution and not its skewness.(LOS 8.k)5.C Normal distributions have kurtosis of three and excess kurtosis of zero.(LOS8.l)6.B The arithmetic mean is appropriate for forecasting single period returns.Thegeometric mean is appropriate for forecasting compound returns over more than a single period.(LOS8.m)Module Quiz9.11.C An event is said to be exhaustive if it includes all possible outcomes.(LOS9.a)2.C Probabilities may range from0(meaning no chance ofoccurrence)through1(which means a sure thing).(LOS9.b) 4.C By the multiplication rule of probability,the joint probability of two events,P(AB),is the product of a conditional probability,P(A|B),and an unconditional probability,P(B).(LOS9.d,9.e)5.C There is no intersection of events when events are mutually exclusive.P(A|B)=P(A)×P(B)is only true for independent events.Note that since A and B are mutually exclusive(cannot both happen),P(A|B)and P(AB) must both be equalto zero.(LOS9.f)6.B P(name1or name2or name3or name4)=1/800+ 1/800+1/800+1/800=4/800=0.005.(LOS9.f)7.B One or the other may occur,but not both.(LOS9.f) Module Quiz9.21.C Two events are said to be independent if the occurrence of one event does not affect the probability of the occurrence of the other event.(LOS9.g)2.B The three outcomes given for economic growth are mutually exclusive and exhaustive.The probability that economic growth is positive but less than3%is100%–25%–25%=50%.Using the total probability rule,theprobability that the share price increases is(80%)(25%)+ (40%)(50%)+(10%)(25%)=42.5%.(LOS9.h,9.i)3.A Covariance may have any value.Correlation is bounded by–1and 1.(LOS9.k)Module Quiz9.31.B E(X|Y=1)=(0.2)(0)+(0.4)(5)+(0.4)(10)=6E(X|Y=2)=(0.1)(0)+(0.8)(5)+(0.1)(10)=5E(X)=(0.3)(6)+(0.7)(5)= 5.30(LOS9.l)2.B Expected value=(1/4)(1+2+3+4)= 2.5Variance=(1/4)[(1−2.5)2+(2−2.5)2+(3−2.5)2+(4−2.5)2]=1.25Note that since each observation is equally likely,each has25%(1/4)chance of occurrence.(LOS9.l)3.A(LOS9.l)4.C This is an application of Bayes’formula.As the tree diagram below shows,the updated probability that earnings per share are greater than$2is(LOS9.n)5.B(LOS9.o)6.B Since the order of the top three finishers matters, we need to use the permutation formula.(LOS9.o)Module Quiz10.11.B Time is usually a continuous random variable;the others are discrete.(LOS10.a)2.B For a continuous distribution p(x)=0for all X; only ranges of value of X have positive probabilities. (LOS10.a)4.B From the table.(LOS10.d)5.C(0.04+0.11+0.18+0.24+0.14+0.17)=0.88(LOS 10.c)6.B(0.14+0.17+0.09+0.03)=0.43(LOS10.d)7.C(0.18+0.24+0.14+0.17)=0.73(LOS10.d)8.A0+1(0.11)+2(0.18)+3(0.24)+4(0.14)+5(0.17)+ 6(0.09)+7(0.03)= 3.35(LOS10.d)9.A There may be any number of independent trials,each with only two possible outcomes.(LOS10.e)10.B With only two possible outcomes,there must be some positive probability for each.If this were not the case, the variable in question would not be a random variable, and a probability distribution would be meaningless.It does not matter ifone of the possible outcomes happens to be zero.(LOS10.e)13.C Success=passing the exam.Then,E(success)=np= 15×0.4= 6.(LOS10.f)14.C F(x)is the cumulative probability,P(x<20)here. Because all the observations in this distribution are between4and10,the probability of an outcome less than 20is100%.(LOS10.h)Module Quiz10.21.A Normal distributions are symmetrical(i.e.,have zero skewness)and their kurtosis is equal to three.(LOS10.i)2.B To describe a multivariate normal distribution,we must consider the correlations among the variables as well as the means and variances of the variables.(LOS10.j)3.C z=(100−175)/25=–3,F(–3)=1−0.9987= 0.0013.(LOS10.k)4.C1−F(2),where F(2)equals0.9772.Hence,1−0.9772= 0.0228.(LOS10.k)5.C1−F(–1)=F(1)=0.8413.There is an84.13% probability that a randomly chosen income is not more than one standard deviation below the mean.(LOS10.k)6.B This is true by the formula for z.(LOS10.l)7.C By the symmetry of the z-distribution and F(0)=0.5. Half the distribution lies on each side of the mean.(LOS10.l)Module Quiz10.31.C SFR=(18−4)/40=0.35is the largest value.(LOS 10.m)2.A SFR=(5−0)/8=0.625is the largest value.(LOS 10.m)3.A Using the tables,the cdf for–1.3is9.68%,which is the probability of returns less than2%.(LOS10.m)4.B A lognormally distributed variable is never negative. (LOS10.n)5.B ln(23/20)=0.1398(LOS10.o)6.B ln(2)=0.6931(LOS10.o)7.B Monte Carlo simulation involves modeling asset prices or returns by generating random values for the risk factors that affect the price of a security.(LOS10.p,9.q)Module Quiz11.11.B In a simple random sample,each element of the population has an equal probability of being selected. Choice C allows for an equal chance,but only if there are 100elements in the population from which the random sample is drawn.(LOS11.a)2.C Suppose you have a population of10,000employees.If you take100samples of50employees each,the distribution of the100sample means is the sampling distribution.(LOS11.a)3.C An example might be the difference between a particular sample mean and theaverage value of the overall population.(LOS11.b)4.B The sampling error is the difference between the population parameter and thesample statistic.(LOS11.b)5.B Cross-sectional data is a set of data that are all collected as of the same point intime.(LOS11.c,11.d)6.C Sample sizes of30or greater are typically considered large.(LOS11.e)11.A Efficiency,consistency,and unbiasedness are desirable properties of an estimator.(LOS11.g)Module Quiz11.21.A As the degrees of freedom get larger,the t-distribution approaches the normal distribution.As the degrees of freedom fall,the peak of the t-distribution flattensand its tails get fatter(more probability in the tails—that’s why,all else the same,the critical t increases as the df decreases).(LOS11.i) 4.C Use the t-statistic atα/2and n–1degrees of freedom when the populationvariance is unknown.While the z-statistic is acceptable when the sample size islarge,sample size is not given here,and the t-statistic is always appropriate underthese conditions.(LOS11.j)5.A When the sample size is large,and the central limit theorem can be relied uponto assure a sampling distribution that is normal,either the t-statistic or the z-statistic is acceptable for constructing confidence intervals for the populationmean.The t-statistic,however,will provide a more conservative range(wider)at agiven level of significance.(LOS11.j)8.A The primary example of look-ahead bias is using year-end financial information in conjunction with market pricing data to compute ratios like the price/earnings(P/E).The E in the denominator is typically not available for30–60days after the end of the period.Hence,data that was available on the test date(P)is mixed with information that was not available(E).That is,the P is “ahead”of the E.(LOS11.k)9.C Mutual fund performance studies are most closely associated with survivorship bias because only the better-performing funds remain in the sample over time.(LOS11.k) Module Quiz12.11.C To test whether the population mean is greater than 20,the test would attempt to reject the null hypothesis that the mean is less than or equal to20.The null hypothesis must always include the“equal to”condition. (LOS12.a)2.C Rejecting the null when it is actually true is a TypeI error.A Type II error is failing to reject the null hypothesis when it is false.The significance level equals the probability of a Type I error.(LOS12.b,12.c)3.C A Type I error is rejecting the null hypothesis when it’s true.The probability of rejecting a false null is [1−Prob Type II]=[1−0.60]=40%,which is called thepower of the test.A and B are not necessarily true,since the null may be false and the probability of rejection unknown.(LOS12.c)4.A The power of a test is1−P(Type II error)=1−0.15 =0.85.(LOS12.d)Module Quiz12.21.B With a p-value of3%,the manager can reject the null hypothesis(that abnormal returns are less than or equal to zero)using a significance level of3%or higher. Although the test result is statistically significant at significance levels as small as3%,this does not necessarily imply that the result is economically meaningful.(LOS12.e,12.f)2.A With a small sample size,a t-test may be used if the population is approximately normally distributed.If the population has a nonnormal distribution,no test statistic is available unless the sample size is large.(LOS12.g)3.B This is a two-tailed test with14–1=13degrees of freedom.From the t-table, 2.160is the critical value to which the analyst should compare the calculated t-statistic.(LOS12.g)Module Quiz12.31.C When the variances are assumed to be unequal,we just calculate thedenominator(standard error)differently and use both sample variances to calculate the t-statistic.(LOS12.h) 2.A Since the observations are likely dependent(both related to market returns),a paired comparisons(mean differences)test is appropriate and is based on a t-statistic.(LOS12.h,12.i)3.C There is no consistent relationship between the mean and standard deviation of the chi-square distribution or F-distribution.(LOS12.j)4.B The F-test is the appropriate test.(LOS12.j)5.C A test of the population variance is a chi-square test.(LOS12.j)Module Quiz13.11.A Technical analysis assumes persistent trends and repeating patterns in market prices can be used to forecast price behavior.Technical analysts believe prices reflect supply and demand,but that buying and selling can be motivated by both rational and irrational causes.Volume,along with price,is important informationto a technical analyst.(LOS13.a)2.B Candlestick charts show the open,high,low,and close for each trading period.Line charts use only the closing price.Point-and-figure charts do not necessarily show distinct trading periods.(LOS13.b)3.A The downtrend reached a support level where buying demand sustained the price.A resistance level is a price at which selling pressure emerges that stops an uptrend. The change in polarity principle holds that breached support levels become resistance and breached resistance levels become support.With no information given on the stock’s history,we cannot determine whether$30had once been a resistance level.(LOS13.c)4.A Bollinger bands are based on the standard deviation of prices over some number of the most recent periods.An RSI is based on the sums of positive and negative price changes during a period.An ROC oscillator is based on the difference between the most recent closing price and the closing price a given number of periods earlier.(LOS13.e)5.B The RSI is calculated from the ratio of total priceincreases to total price decreases over a chosen number of days,then scaled to fluctuate between0and100using the formula RSI=100−[100/(1+ratio of increases to decreases)].Stochastic oscillators are based on the highest and lowest prices over a chosen number of days. MACD oscillators are calculated based on exponentially smoothed moving averages.(LOS13.e)6.B“More bullish”means investors expect prices to increase in the near term.Increasing margin debt suggests investors are bullish and buying aggressively.Increases in put volume relative to call volume,or in the number of shares sold short,indicate bearish investor sentiment. (LOS13.e)7.C The Kondratieff wave is a54-year cycle.(LOS13.f)8.C The value 1.618is the ratio of large consecutive Fibonacci numbers.Technical analysts who employ Elliott wave theory frequently use Fibonacci ratios to setprice targets.(LOS13.g)9.B If the relative strength ratio(stock price/ benchmark value)increases,the stockis outperforming the benchmark stock or index against which it is being measured.This does not imply that thestock is increasing in price;if the stock price decreases but the benchmark decreases by a larger percentage,the ratio will increase.Volume is not an input into a relative strength ratio.(LOS13.e,13.h)。

  1. 1、下载文档前请自行甄别文档内容的完整性,平台不提供额外的编辑、内容补充、找答案等附加服务。
  2. 2、"仅部分预览"的文档,不可在线预览部分如存在完整性等问题,可反馈申请退款(可完整预览的文档不适用该条件!)。
  3. 3、如文档侵犯您的权益,请联系客服反馈,我们会尽快为您处理(人工客服工作时间:9:00-18:30)。

CFA各科试题集及解答--QUIZ 6六 Investment Tools: Economics: Global Economic Analysis1.A: Gaining from International Trade Question ID: 12842Which of the following will determine the pattern of international trade?Comparative advantage. A.Profit margin advantage. B.Absolute advantage. C.Lowest total cost advantage. D.AComparative advantage is the ability to produce a good at a lower opportunity cost than otherscan produce it.Question ID: 12844Which of the statements about absolute advantage is TRUE? Absolute advantage:refers to the lowest opportunity cost to produce a product. A.is when both trading partners benefit. B.is when a nation trades for goods for which they are the high-opportunity C. cost producer.is when a nation can produce more output with the same resources than D. any other nation.DBy definition.Question ID: 12801Which type of advantage determines the pattern of trade in the world?Absolute advantage. A.Comparative advantage. B.Advantages due to tariffs and quotas. C.Economic advantage. D.BComparative advantage is the ability to produce a good at a lower opportunity cost than otherscan produce it. According to the law of comparative advantage,trading partners are bothbetter off if they specialize in the production of goods for whichthey are the low-opportunitycost producer and trade for goods for which they are the high-opportunity cost producer.Question ID: 12807The table below outlines the possible tradeoffs of producing unitsof wine and wheat for bothCountry A and Country B.Country A Country Bwine wheat wine wheat0 8 0 56 units 4 4 units 312 0 10 0Using the table above, the law of comparative advantage suggests that:Country A would not gain from trade, because it has an absolute advantage A. in the production of both goods.neither country would gain from trade, even if the costs for transporting the B. products were zero.both countries would gain if Country A traded wheat for Country B's wine. C.both countries would gain if Country A traded wine for Country B's wheat. D.CCountry A has an absolute advantage in both wheat and wine becauseit can produce more ofboth than country B. Country B has a comparative advantage in producing wine because it canproduce either 5 wheat or 10 wine which is a ratio of 1 to 2 whereas country A can produce 8 wheat or 12 wine which is a ratio of 1 to 1.5. Because of country B's comparative advantage in producing wine both countries would benefit if A used its absolute advantage in producing wheat in exchange for B's comparative advantage in wine.Question ID: 12805The law of comparative advantage explains why a nation will benefit from trade when it:exports goods for which it is a high-cost producer, while importing those for A. which it is a low-cost producer.exports more than it imports. B.imports more than it exports. C.exports goods for which it is a low-cost producer, while importing those for D. when it is a high-cost producer.DComparative advantage is the ability to produce a good at a lower opportunity cost than others can produce it. When trading partners specilize in producing products for which they have a comparative advantage, costs are minimized, output is greater, and both trading partners benefit.Question ID: 12856Which of the following is least likely to affect exchange rates? Differential:spending by firms. A.inflation rates. B.interest rates. C.income growth. D.AThe main determinant of exchange rates is the supply and demand for a currency, which is determined by the difference between the two countries in their: income growth, inflation rates, and interest rates.Question ID: 24992Country A has a comparative advantage in the production of Product F. Assume the current world price is greater than the domestic market-clearing price. If the world price is allowed to prevail in the domestic market, which of the following with respect to Product F is TRUE?The quantity supplied is lower. A.The quantity demanded is higher. B.Revenues of producers are lower. C.The quantity supplied is higher. D.DAt prices above the domestic equilibrium price, the domesticquantity demanded will be lower,and the quantity supplied will be higher.Question ID: 12855A country permits free international trade, and the world marketprice for corn is higher thanthe domestic price would be if the economy were closed. What is the likely impact of the open economy on the price of corn in the domestic market and quantity of corn supplied to the non-domestic market?Price QuantityA. lower lowerPrice QuantityB. lower higherPrice QuantityC. higher higherPrice QuantityD. high lowerCThe direct impact of the open economy on the price of corn in the domestic market is likely to be a higher price due to the greater total demand for the corn. The quantity of corn supplied to the non-domestic market is also likely to be higher because the world economy issupplyingcorn to the market.Question ID: 24994Which of the items below is NOT a reason why nations adopt trade restrictions? To:protect industries that are highly sensitive to national security. A.prohibit foreign firms from increasing market share by sellingproducts B. below cost.protect industries in which they have a comparative advantage. C.protect industries while they are developing. D.CIf a particular country enjoys a comparative advantage in aparticular industry, no protection isneeded.Question ID: 12857Which of the following arguments for trade restrictions have loweror questionable validity?Tariffs are used to prohibit foreign firms from selling products in the country A. at below cost in an attempt to gain market share.Some industries are highly sensitive to national security and their products B. should, therefore, remain in the country.Infant industries should be protected for a time while they develop and C. reduce costs.Trade restrictions create domestic jobs. D.DThe reading states that while all other explanations are partially correct, "Trade restrictionscreate domestic jobs" is at best of questionable validity.Question ID: 24996The anti-dumping argument in favor of trade restrictions is the argument the restrictionsshould be imposed to:discourage foreign firms from engaging in price competition. A.prevent foreign firms from dumping unwanted products in domestic B. markets.discourage foreign firms from exporting products of inferior quality.C.prevent foreign firms from selling their product below cost. D.DThe anti-dumping argument is that restrictions should be used to prohibit foreign firms fromincreasing market share by selling products below cost. Question ID: 12861Who is the ultimate loser when trade restrictions are established?Foreign producers. A.Foreign governments. B.Domestic consumers. C.Domestic producers. D.CConsumers lose due to the higher prices of goods.Question ID: 12863If the U.S. auto producers were prohibited from exporting their products abroad, which ofthe following would most likely occur?Auto prices in the US would rise. A.The incomes of US auto makers would rise. B.U.S. consumers would gain at the expense of foreign producers. C.Auto prices in the U.S. will fall, but U.S. citizens would have less foreign D. exchange with which to purchase foreign assets abroad.DIf U.S. auto producers could not export their products abroad their price would fall because demand would be reduced.? With the decreased demand and associated decreased production in autos this could have a ripple effect through the economy of one or more of the following: less dollars flowing into the U.S. because of decreased exports, decreasedproduction in the large auto industry could be a factor leading to a recession,? decreasing prices would limit salaries for auto workers.?All of these factors would inhibit the purchase of foreign products by U.S. citizens.?Question ID: 12865Who benefits from a quota?The government. A.Foreign producers. B.Foreign consumers. C.Domestic producers. D.DQuotas restrict the supply of imported goods, which increases the price domestically benefiting domestic producers.Question ID: 12866Which of the following is NOT a valid reason why nations adopt trade restrictions? Nations adopt trade restrictions to:protect national defense. A.prevent firms from selling products in a country at below cost. B.protect infant industries. C.protect jobs. D.DIn the long run, trade restrictions cannot protect the net number of jobs in a country. The number of jobs protected by import restrictions will be offset by the number of jobs lost in the import/export industry.Import/export firms will be unable to sell the overpriced domestic product abroad or import and sell the lower priced restricted foreign-made product.Question ID: 24999Which of the following reasons for a nation to adopt trade restrictions is the least valid? The:anti-dumping argument. A.national defense argument. B.job protection argument. C.infant industry argument. D.CThe number of jobs protected by import restrictions will be offset by jobs lost in the in the import/export industries. In the long run, trade restrictions cannot protect the net number ofjobs in a country.Question ID: 12867"Import quotas will create jobs, increasing the employment level of a nation." Economic analysis indicates that this statement is in the:Short Run Long RunA. false trueShort Run Long Run B.true falseShort Run Long RunC. true trueShort Run Long RunD. false falseBThe argument that trade restrictions protect jobs is of questionable validity. First, trade restrictions prevent trading partners from developing the purchasing power needed to buy import goods from the protected country, thus depressing the country's own export industry. Second, the higher price of the protected domestic goods dampens domestic purchasing power, taking sales away from other domestic products. Third, the jobs that would have been created in the import industry are never created.The number of jobs protected by import restrictions will be offset by jobs lost in the import/export industry.Question ID: 12806The following table illustrates the domestic prices of items of similar quality - shoes,watches, and electric motors - in the United States and Israel.United States IsraelItem (dollars) (pounds)Shoes 20 40Watches 40 90Electric Motors 80 300Assuming that the exchange rate for the Israeli pound is 2.50 perU.S. dollar and that transportation costs are zero, which goods will the U.S. tend to export to Israel?Shoes, watches, and electric motors. A.Shoes and watches only. B.Shoes only. C.Electric motors only. D.DThe key to this question is to convert the Israeli pound to dollars. 40/2.5 = $16 shoes, 90/2.5 = $36 watches, 300/2.5 = $120 electric motors, the electric motors are the only item that can be sold at a higher price in Israel than the U.S. therefore it would be the only product exported from the U.S. to Israel.Question ID: 12858Which of the following is NOT a valid reason for a country to adopt trade restrictions?To prohibit foreign firms from selling products in a country atbelow cost in A. an attempt to gain market share.To create jobs and protect the net number of jobs within a country. B.To protect infant industries. C.For national security reasons. D.BIn the long run, trade restrictions cannot increase or protect thenet number of jobs in a country. Trade restrictions prevent trading partners from developing the purchasing power needed to buy your import goods thereby depressing your export industry reducing those jobs that would have been created through increased exports.Question ID: 12803The following chart indicates the production possibilities of food and drink per day in Country A and Country B.Units of Output Per DayCountry A Country BFood 9 5Drink 7 5Which of the following statements is TRUE?Mutual gains could be realized from trade if A specialized in drink A. production and B specialized in the food production.Since B workers can produce more of food and drink than A workers,no B. gains from trade are possible.B is the low-cost producer of both food and drink. C.Mutual gains could be realized from trade if A specialized in food D. production and B specialized in drink production.DMutual gains could be realized from trade if A specialized in food production and B specialized in drink production. The reason centers on comparative advantage. Country A must give up th 7/9unit of drink toproduce one unit of food. Country B must give up 1 unit of drink to produce one unit of food. Therefore, the opportunity cost of producing food is greater for B than for A. If B produces 5 units of drink and A produces 9 units of food, each country can have 4.5 units of food and 2.5 units of drink and they will both benefit compared to other possible combinations of production.Question ID: 12808The table below outlines the possible tradeoffs of producing beer and cheese for Germany and Holland.Germany HollandCheese Beer Cheese Beer0 10 0 65 0 4 0Which of the following statements is TRUE?Germany would not gain from trade, because it has an absolute advantage A. in the production of both goods.Neither country would gain from trade. B.Both countries would gain if Germany traded beer for Holland's cheese. C.Both countries would gain if Germany traded cheese for Holland's beer. D.CQuestion ID: 12809Why does trading with low-wage countries not depress wage rates in high-wage countries?Comparative advantage and worker productivity determine who should produce what, given other factors of production. A high-wage country withhigher worker productivity in one sector can produce those goods more A.efficiently than the low-wage country and trade with a low-wage countrythat has higher worker productivity in another sector.The law of supply and demand for labor will provide a floor below whichwages cannot fall in either country and therefore, neither country's workers B.lose in terms of their respective wages.Because low wage countries buy products from high wage countries C. thereby increasing demand for workers and maintaining the wage level.Because if the wage rates were to start falling, the government of the highwage country would impose tariffs on the products of the low wage country D.to prevent the wages from falling.AQuestion ID: 12868Which of the following is a reason why trade with low-wage countries does NOT depresswage rates in high-wage countries?When each country produces goods for which it has a comparative A. advantage, both countries benefit.Low-wage countries have an advantage in high tech manufacturing. B.High hourly wage rates mean high per unit labor costs. C.High-wage countries have an advantage in labor-intensive goods. D.AWhen both countries produce the goods in which they have an advantage, total output and theavailability of goods will increase.Question ID: 12860Who benefits least from tariffs?The domestic government. A.Domestic producers. B.Domestic consumers. C.The foreign government. D.CA tax imposed on imports is called a tariff, which benefits domestic producers and domestic governments. Domestic consumers lose through higher prices, less choice of products, and lower quality products.1.B: International Finance and the Foreign Exchange MarketQuestion ID: 12871In a floating exchange rate system, if there is an excess demand for: British pounds by the Belgians, Belgians will lower their interest rates so as A. to enable their citizens to borrow more easily in orderto buy British goods.Swiss francs by Germans, Germans will sell marks and buy francs.This will B. cause the mark to depreciate relative to the franc.Italian liras by Spaniards, Italians will lower their interest ratesso as toenable Spaniards to buy Italian goods on credit and satisfy their demand for C.Italian products.German goods by Americans, Americans will have to sell more goods to D. Germans so as to be able to buy more German goods.BIn a floating exchange rate system, exchange rates between countries are based on the demand and supply of currencies relative to each other.If Germans demand francs, they will sell marks and buy francs in exchange, thus depressing their own currency. The franc will appreciate relative to the mark. Other choices are incorrect because they are not based on thesupply and demand argument underlying floating exchange rates.Question ID: 25000In a flexible exchange rate system, exchange rates are determined by: supply and demand in the market for the currency. A.governmental fiat. B.trade restrictions. C.the total value of the country's gold reserves. D.AExchange rates are determined by supply and demand. Britishimporters needing dollars to purchase U.S. goods will buy U.S. dollars and sell British pounds. British exporters needing to convert dollars to pounds will sell dollars and buy pounds.Question ID: 12870A U.S. tourist planning to visit Germany exchanges $500 for German marks at a rate of $.65/DM, but her trip is cancelled. When she exchanges her marks for dollars, she receives $577. How many DM did she receive when making the initial change, what was the DM/$ exchange rate, and what was the $/DM exchange rate?Marks Received DM/$ Exchange Rate $/DM Exchange Rate A. 325 1.09DM/$ $1.15/DMMarks Received DM/$ Exchange Rate $/DM Exchange Rate B. 549 0.65DM/$ $0.65/DMMarks Received DM/$ Exchange Rate $/DM Exchange Rate C. 769 1.54DM/$ $0.75/DMMarks Received DM/$ Exchange Rate $/DM Exchange Rate D. 811 1.62DM/$ $1.47/DMC$500 x 1DM/$.65 = 769 marks1/.65 = 1.54 DM/$$577/769DM = $0.75/DMQuestion ID: 12880The factor most likely to cause a nation's currency to appreciate on the foreign exchange market is:an increase in the nation's foreign investment (assets purchasedfrom A. foreigners).an increase in real interest rates in other countries. B.an increase in exports relative to imports. C.rapid domestic inflation. D.CDemand for foreign currencies comes from demand for things produced by foreigners. The supply of U.S. dollars on the foreign exchange market comes from non-Americans buying things from Americans. If imports decrease and exports increase there will be an increased demand for a nation's currency because foreign countries will be purchasing more goods from abroad, thus appreciating the country's currency.Question ID: 12874Which of the following is least likely to cause a domestic currency to depreciate?A high inflation rate. A.Faster growth of imports relative to exports. B.Domestic real interest rates are less than those abroad. C.Slow growth of income relative to one's trading partners. D.DSlow growth of income relative to one's trading partners will cause imports to lag behind exports. When the demand for a country's exports increases the demand for their currency also increases causing their currency to appreciate.Question ID: 12876Under a system of flexible exchange rates, which one of thefollowing is more likely to cause a nation's currency to appreciate on the foreign exchange market?A decrease in domestic interest rates. A.An increase in foreign interest rates. B.A domestic inflation rate lower than the nation's trading partners.C.A domestic inflation rate higher than the nation's trading partners.D.CIf a nation's trading partners prices are increasing twice as fast as the domestic country A, then foreign citizens will increase their demand for A's goods. This increased demand will appreciate country A's currency making country A's goods more expensive offsetting the effects of inflation.Question ID: 12888If incomes in U.S. are rapidly increasing compared to those in Mexico, how will the value of the U.S. dollar and Mexican peso move relative to each other?U.S. dollar Peso A. appreciate depreciateU.S. dollar Peso B. depreciate appreciateU.S. dollar Peso C. no change no changeU.S. dollar Peso D. depreciate no changeBRapid growth of U.S. incomes relative to incomes in Mexico will stimulate imports from Mexico causing an increased demand for the peso. The increased demand for pesos will cause the peso to appreciate relative to the dollar.Question ID: 12887Which of the following would cause a nation’s currency to depreciate?Slow growth of income relative to one’s trading partners. A.Domestic real interest rates that are lower than those of other countries. B.Discouraging imports and encouraging exports. C.A rate of inflation that is lower than that of one’s trading partners. D.BOther factors that will cause a nation's currency to depreciate are: rapid growth of incomerelative to one's trading partners that stimulates imports relative to exports and a rate ofinflation that is higher than that of one's trading partners.Question ID: 12886Which of the following factors does NOT affect foreign exchange rates?Differential inflation rates. A.Income growth. B.Real interest rates. C.The government sets a price floor for the price of wheat. D.DBy definition.Question ID: 25001In balance of payments accounting, the net inflow of debt and equity investment funds intothe country appears in the:official reserve account. A.current account. B.capital account. C.current account adjusted for capital depreciation. D.CThe capital account measures the flow of debt and equity investment funds into and out of thecountry.Question ID: 12890What is the difference between the balance of merchandise trade and the balance of payments?The value of goods, services, and unilateral transfer is included in thebalance of merchandise trade, while the balance of payments includes A.both current accounts and capital account transactions.The value of goods imported and exported is included in the balance ofmerchandise trade, while the balance of payments includes onlycapital B.account transactions.Only the value of goods imported and exported is included in the balance ofmerchandise trade, while the balance of payments includes the value of all C.payments to and receipts from other nations.Balance of merchandise trade and balance of payments are different terms D. used to describe the same international exchange transactions.CBy definition.Question ID: 25002If a nation is running a deficit in the current account, the sum of the capital account and the official reserve account must be zero. A.positive. B.in deficit. C.negative. D.BThe balance of payments equation is: Current account balance +capital account balance + official reserve account balance = 0 If the current account balance is in deficit, the others must be positive for the sum of these balances to be zero.Question ID: 25004An unanticipated shift to a federal government surplus would cause the capital account tomove to:deficit and the current account to move to deficit. A.surplus and the current account to move to deficit. B.deficit and the current account to move to surplus. C.surplus and the current account to move to surplus. D.CAn unexpected shift to a larger budget surplus would cause a decrease in aggregate demand and a reduction in domestic interest rates. This reduced demand discourages imports, which moves the current account toward surplus. The lower interest rates will encourage domesticinvestment to leave the country and discourage foreign investors form investing in the U.S. The capital account will move toward deficit.Question ID: 25003An unexpected increase in the growth rate of the money supply would: have no effect on exchange rates in the short run. A.cause real interest rates and aggregate employment to increase. B.cause real interest rates to fall, causing a depreciation of the dollar. C.cause real interest rates to rise, causing an appreciation of the dollar. D.Cp>Unanticipated shifts to an expansionary monetary policy would lead to a more rapid economic growth, an unexpected increase in inflation, and lower real interest rates. The more rapid economic growth would lead to an increase in demand for imports. The higher rate of inflation makes domestic goods more expensive, reducing exports. Lower real interest rates reduce foreign investment. These factors increase the demand for foreign currencies and reduce the demand for the dollar, causing the dollar to depreciate.Question ID: 12892An unanticipated shift to an expansionary monetary policy will NOT lead to?more rapid economic growth, an accelerated inflation rate, and lower real A. interest rates.more expensive domestic products, which reduces exports. B.reduced foreign investment. C.an appreciating domestic currency. D.DAn unanticipated expansionary monetary policy will lead to all ofthe others except an appreciating domestic currency. Higher inflation will increase domestic products prices and make them unattractive to foreigners. As a result foreigners will reduce their demand for domestic products and will not demand the domestic currency as much as before. Coupled with declining foreign investment, which will also lead to reduced demand for the domestic currency the domestic currency valuewill fall relative to other currencies.Question ID: 25005Current account deficits are:not an indication of a nation's economic health. A.usually accompanied by a deficit in the capital account. B.an indication that the economy is growing rapidly. C.the result of a country importing less than it exports. D.AA current account deficit occurs when a country imports more than it exports, and it is not an indication of economic health. There is no requirement that the current account balance be zero, in surplus or in deficit.Question ID: 25006A current account surplus:is not an indication of a nation's economic health. A.is an indication of the strength of the economy. B.occurs when a country exports less than it imports. C.is an indication of significant foreign investment in the domestic market. D.AA current account surplus occurs when a country exports more than it imports, and it is not an indication of economic health. There is no requirement that the current account balance be zero, in surplus or in deficit.Question ID: 12894Which of the following statements is TRUE?A nation's current account surplus or deficit is a good measure of the health A. of its economy.Capital inflows from foreigners are not bad even if the foreigners buy up B. domestic real estate, domestic industries and own other productive assets.Running a deficit in the current account balance simply means a country C. imports more than it exports, but a country can do this only for a short time.Countries that run current account deficits also run capital account deficits. D.B。

相关文档
最新文档